Sie sind auf Seite 1von 76

1

November Questions 2018


Last updated 16/11/2018 10:01 Am by Aziz J*

1/ Heart anomaly LEAST associated infective endocarditis ?


Answer: ​VSD or ASD
Medscape : ASD 16% , VSD 15%
Uptodate : VSD most common 32% .
Peda notes : the answer was ASD

2/ Antidepressant less likely to cause sexual dysfunction?


Answer : ​bupropion

3/ Antipsychotic less likely to cause TD?


Answer: ​Clozapine

4/ Alcohol withdrawal treatment ?


Answer :​ Chlordiazepoxide/Diazepam

5/ Alzheimer’s e rash and angular cheilitis asked about vitamins?


Answer: ​b12

6/ Hbsag -ve when to repeat test?


Answer: ​The antigens are detectable 4–10 weeks after exposure to HBV ( 1-2 months)

7/ Which causes loss of colour vision?


Answer:​ optic atrophy

8/ Scenario of sob & cough with « Crescentic lucency surrounding the peripheral aspect
of the mass on x-ray what is the most likely diagnosis ?
answer : ​aspergillosis

9/ Which pneumocystis disease precipitates TB ?


Answer:​ silicosis

10/ ​Bacteroides fragilis ​ treatment ?


Answer: ​metronidazole then go for Carbapenems ( clindamycin associated with 60 %
Resistance )

11/ Complications of temporal arteritis?


2

Answer: ​complete loss of vision​ ​( blindness)

12​​/ ​Vasectomy case pt wanted to reverse it and there were antisperm antibodies asked
about mechanism​​?
Answer: ​Antigen release

13/ Prognosis of breast ca ?


Answer : ​axillary metastasis
14/ The same CF case with same choices
A. ½
B. 2/4
C. ⅔
D. 1/3
Answer : ​C
15/ Research Question same as written in old files

16/ ​threatened abortion features? 


Answer​ : - ​ little of bright red bleeding and the cervix is closed 
( bleeding before 20 weeks and OS is closed )  
17/ fourth lumbar spine stone asking where is this stone located? 
Answer​ : ​Ureter L3-4 
18/ Indian ink test positive in a patient with HIV what is the most likely 
diagnosis ?  
Answer: c​ ryptococcus 
19/ Taste in anterior ⅔ of tongue ? 
Answer: ​Facial 
20/ TOF feature ? 
Answer ​Pulmonary stenosis VSD overriding aorta Right ventricle hypertrophy 
21/ 13 C urea breath test diagnosis ?  
Answer: ​H-pylori 
22/rRNA located in ? 
Answer ​nucleolus ( 2R = 2L)  
23/ septic arthritis treatment ? 
Answer: ​Aspiration + IV AB 
24/ Fracture with skin ulceration? 
Answer: ​debridement and AB

25/ Fracture in tibia what is the nerve involved ?  


Answer: ​tibial nerve
● fracture at fibular neck > common peroneal (fibular) nerve injury .
3

● - Loss of eversion of foot > superficial peroneal injury


● - Loss of dorsiflexion of foot (foot drop) and sensation of web between 1st and 2nd toe >
deep peroneal nerve injury
● - Loss of both eversion and dorsiflexion (foot drop) > common peroneal nerve injury
● - Loss of inversion of foot > tibial nerve injury
26/ Case about pyoria and fever in testis what is the diagnosis? 
Answer ​Epidydemorchitis 
27/ parasite increase which IL ? 
Answer: ​IL-5 
28/ Chronic Severe pancreatitis sign ? 
Answer: ​Cullen, grey turner, fox. Necrotizing pancreatitis  
29/ where enzyme of pancreas leake  
Answer: ​Omental bursa 
30/ site of celiac ? 
Answer: ​Proximal small bowel 
31/ Site of meckel's ? 
Answer: ​Lower part of ilium or terminal  
32/ Treatment of celiac ?  
Answer: ​Gluten free diet 
33/side effect of corticosteroids ? 
Answer: ​growth retardation  
34/ Which cells secrete melanin ? 
Answer: ​Corticotroph secretes ACTH and melanin 
35/ bee sting treatment ? 
Answer: ​If said in question SOB >> epinephrine
If said urticaria.>> Antihistamine
If said nothing >> remove stinger
36/Trauma and receptor activated what will happened  
Answer ​Venous capatinous  
37/ dog bites caused by ?  
Answer: ​Polymicrobial 
38​/Commonest infections by animals ?  
Answer :​ Pasteurella multocida 
39/A patient had gastrectomy and needs to take vitamin B12 for life. Which 
cells are responsible? 
Answer: ​Parietal cell 
40/ Down syndrome comments cardiology abnormality 
Answer: ​AVSD  
41/ decrease BP with standing without any other symptoms? 
4

Answer: ​Orthostatic hypotension 


42/millstone from prone to supine what age? 
Answer:​ 4 months  
43/millstone no assist pincer grasp 
Answer:​ 12 months  
44/ delay millstone at age of 12 months 
Answer​ :​Crawl or sit without support 
45/herpetic keratitis pic  
46/ trauma case there is Decreased breath, hypotension and tachycardia,what 
is the initial management ? 
Answer: ​Needle decompression 
47/ most common LN enveloped in breast cancer ? 
Answer: ​Upper outer quadrant (superior lateral) 
48/ what is the most common site of breast cancer ? 
a. Anterior
b. Lateral
c. Postero
d. Medial  
Answer: ​A  
49/ most common cause of hospital infection ? 
Answer: ​Hand of the staff  
50/ GDM OR was 2  
51/ MRSA 66:1 ( non exposed was 500) 
52/ phrenic nerve found in ? 
Answer: : ​Anterior to anterior scalene muscle 
53/ which of the following acidophilic ? (fungal) 
Answer: ​Actinomyces acidophilus 
54/ LP Case  
55/ Gas gangrene treatment  
Answer: ​penicillin and clindamycin, and debridement
56/ methicillin sensitive ?  
Answer​: C
​ loxacillin 
57/ mode of inheritance medellin ? 
Answer​ : ​thalassemia  
58​/ SCA case what limb pain what is the immediate to do he had large 
splenomegaly ?  
Answer:​ IV fluid first (isotonic)  
5

59/A patient came to the emergency department with acute eye pain and 
headache, nausea and vomiting. The patient is a known case of acute 
glaucoma? 
Answer​ : ​acetazolamide and pilocarpine  
60/incidence of prolonged labour ?  
A. 5.15
B. 20.25
C. 30.4  
Answer​: ​10-15 % Although Uptodate says 20% 
61/ Mechanism of Diarrhea 
Answer: ​Decreased absorption 
62/ Case of seborrheic dermatitis ? what's treatment ?  
Answer: ​ketoconazole, selenium sulfide ( if not in the option go for tacrolimus)  
63/ definition of innate immunity? 
Answer: ​Innate immunity is something already present in the body. Adaptive immunity is created
in response to exposure to a foreign substance. 
64/ Which Interleukin deficiency in contraindicated in TB vaccine? 
Answer: ​IL 12 ( INF-Y + interferon alpha)  
65/ A patient complains that his ideas was broadcasting in TV and radio for 4 
months. What is the diagnosis ? 
Answer: ​schizophreniform disorder 
66/When we do trabeculectomy ? 
A. Congenital glaucoma  
B. Open angle glaucoma  
C. Close angle glaucoma  
Answer: ​B  
67/ what is the procedure indicate for ptosis? 
Answer :​ Frontal suspension 
68/Average age of menopause? 
A. 48 
B. 51 
C. 53 
D. 55 
Answer : ​B  
69/ Cellulitis in children what is the organism ? . 
Answer:​ H. Influenza ( purple discoloration) but Less than six months Group A beta hemolytic
streptococcus  
6

70/ SCA case with severe pain in foot and hand what is the type of crisis? 
Answer: ​VOC 
71/ how to differentiate between type 1 and 2 DM  
​ nswer: ​endogenous insulin + C Peptide
A
72/ Tanner stage ( most likely stage 2)  
Answer:
- male : testicular volume between 1.6 and 6 ml; skin on scrotum thins, reddens and
enlarges; penis length unchanged (9–11)
- female : breast bud forms, with small area of surrounding glandular tissue; areola begins
to widen (10–11.5)
73/ effect of ionizing radiation on DNA  
Answer: ​Double strand DNA destruction 
74/ Bilateral mass hydronephrosis and abdominal pain what is the diagnosis ? 
Answer: ​Polycystic kidney ( case need more info )  
75/Toxicity of paracetamol  
Answer: ​N acetylcysteine 
76/ Toxicity of aspirin  
Answer: ​ urine alkalization , Respiratory alkalosis metabolic acidosis 
77/ Toxicity of morphine  
Answer: ​Naloxon 
78​/F
​ emale symptoms of anemia and fever , hb 10 , plt 60
Blood film : fragmented RBCs + spherocyte ,
A. ITP
B. TTP
Answer: ​B
79/ Cancer commonly associated with depression

A. Ovarian
B. Renal
C. Prostate
D. Gastric
Answer: ​Pancreatic cancer
The site of primary cancer also influences rates of depression, with depression being most
common in pancreatic and lung cancers and lowest in invasive skin cancer (2). based in this
article https://www.ncbi.nlm.nih.gov/pmc/articles/PMC4356432/
80/ Common chromosomal abnormality

A. Trisomy 21
B. KF syndrome
C. Trisomy 13
Answer: ​A
7

81/Antiphospholipid syndrome
Answer: ​Primary APS
genetic marker HLA-DR7
Secondary APS
SLE or other autoimmune disorders
Genetic markers: HLA-B8, HLA-DR2, HLA-DR3
Race: Blacks, Hispanics, Asians, and Native Americans
82/ Pt with ASD .. Q about ECG changes
A. Lt ventricular hypertrophy
B. Rt BBB
Answer: ​B

83​​/ Pregnant at term Came with seizure and palpitation .. fetal tachycardia
A. PE
B. IHD
C. Umbilical vein thrombosis
 
Answer: ​amniotic fluid embolism

84/ Most common cause of physical urticaria ?

A. Cold
B. solar
C. cholinergic

Answer : ​dermatographism

85​​/ ​40 y/o female Newly diagnosed with diabetes, what investigation is indicated?

A. HPV
B. HAV
C. pneumococcal

Answer: ​C

HBV ( ​People with diabetes have higher rates of hepatitis B than the rest of the population.) +
pneumococcal . ​Ref CDC
8

86/ Erythematous groin lesion with central clearing ?

Answer : Psoriasis

Closed thing ​Erythema annulare centrifugum

87/ Most common viral meningoencephalitis?

Answer : ​ ​enteroviruses ( Coxsackie or Echovirus groups subtype of enterovirus)

88/ What is explanation of CI 95%?

Answer:​ ​amount of uncertainty associated with a sample population estimate (the mean or
proportion) of a true population.

89/ SIDE effect of beclomethasone on child?

Answer : ​Growth retardation

90/ tomur prevented by vaccine ?

Answer: ​HPV, Adult t cell lymphoma

91/ Effect of metformin on muscle ?

Answer:​ increase peripheral glucose uptake and utilization

92/ Layer of vein that is thick and developed?

Answer: ​Tunica adventitia for veins , tunica media for arteries

93/ What is the meaning of standard deviation?

Answer: ​is a measure that is used to quantify the amount of variation or ​dispersion​ of a set of
data values.

94/ PT with previous mi 10 days ago now with symptoms of cardiac tamponade , what will
happen?

A. St changes
B. prolonged PR
C. AF
9

Answer: ​A
Dressler syndrome occurs 7 days after MI, and present with pericarditis and pericardial effusion
95/ Pt e headache in one eye e nasorrhea & lacrimation, what is prophylaxis?
A. Propranolol
B. Amitriptyline
C. Verapamil
Answer: ​C
96/ Child e central & peripheral cyanosis what is the most common accompanying
symptom?
Answer : ​refuses to feed
97​​/ Child e blunt trauma imaging showed coil spring appearance in terminal 2/3 of
duodenum + high amylase what to do?

A. Gastrojejunal nutrition
B. Parenteral nutrition
C. Exploratory laparotomy
D. Observation

Answer: ​B ( or bed rest )


98/ Symptoms of panic attack, what’s initial management?
Answer :​ breathing into paper bag
99​​/ Pt hep A resolved came after months (i think 6) how liver biopsy will look like?

A. Normal
B. Necrosis periportal
C. Somthing central
Answer : ​A
99/ Eye drops for dryness four times per day, how to apply?

A. 1 in lower fornix
B. 2 in lower fornix
C. 1 in upper
D. 2 in upper
Answer: ​A
100/ Hemoglobin bart’s inpatient with pallor? Dx?

A. Silent thal carrier


B. Homogeneous beta thalassemia
C. Heterogeneous alpha & beta thalassemia
Answer: ​B
101/ Child e epilepsy 3 days ago Fever, congested throat, now free of seizures, rx? Temp
now is 37.9 c
10

A. NSAIDs
B. AMOX
C. Diazepam
D. Some other antiepileptic
Answer: ​B / Acetomorphine
102/ Most specific syx of MDD ?

A. Late morning awakening


B. Fatigue
C. Poor concentration
D. Avoid eye contact
Answer: ​A

103/ Ob/gyn Tree like growth most likely DX ?


Asnwer: ​Condyloma accuminata HPV

104/ Pt from africa e fever malaise and bloody diarrhea?


A. Giardia
B. Entamoeba histolytica
C. Shigella
Answer: ​B
Bloody diarrhea causes: CHESS: Campylobacter, Hemorrhagic e.coli, Entamoeba histolytica,
Salmonella, Shigella
Shigella 1-7 days
Entamoeba 2-4 weeks
Giardia 1-14 days
105/ Muscle unlock the knee?
A. Popliteal
B. Gastrocnemius
Answer: ​A
106/ Researcher, bmi... the same question from fudul?
Answer: ​Skin thickness

107/ Mandibular injury with forceful closer of the jaw , which muscle responsible for that
?
A. Temporalis
B. Masseter
C. Pterygoid
Answer: ​B
Temporalis: elevation and retraction
Masseter: elevation and protrusion
11

108​​/​ Female with lesion on genital, , follicles joint forming painless ulcer , her husband
,also have painless penile ulcer , what is the bacteria, ?
A. Treponem,
B. Haemophilus ducreyi
C. HPV
Answer: ​A
109/ Which drug ,effect the use of OCP?
A. Anticonvulsant
B. Anticoagulant
Answer : ​A
110- Ortho Case , child with elbow injury x -ray attached what is the DX ?
answer : salter harris ( add picture )
111- patient , complain of calf pain while running, which muscle is responsible ?
A. gastrocnemius
B. Solus
Answer: ​A
112/ child with elbow trauma with xray which structure ,affected?
Answer: ​posterior pad.
https://orthoinfo.aaos.org/en/diseases--conditions/elbow-fractures-in-children/
113/case ​ ​about ( VWD) von willebrand.
Answer​ :​​Prolonged BT / Manage bleeding by cryopreciptate
114/ case about ​ ​scleroderma
Answer​ : ​antitopoisomerase antibodies, like anti-scl70 diffuse.
anticentromere antibodies (causing a limited systemic form and the CREST syndrome
115/ ​- cushing features with + glucose in urine , what else the patient will have ?
A. Vitiligo
B. Pigmentation
Answer : ​B
116 / antipsychotic cause eye pigmentation ?
A. Thioguanine
B. thiohexamide
C. Thioridazine
D. Thiothixene
Answer: ​C
117/ dideoxynucleotide use in ?
A. DNA sequencing
B. PCR DNA Replication
Answer : ​A
118/ 3 ECG
119/ ​ ​2 peripheral smear megaloblastic anemia and malaria
120/ latent period of HIV ?
Answer : ​CD4 , Dendritic cells and macrophage
12

121/ Esotropia 25 degree next step


A. fundus exam my answer
B. medial rectus recession
Answer: ​B
122/ End stage liver disease and fungal infection. Treatment ?
Answer: ​Caspofungin

123/ Best test assess asthma severity?


Answer: ​FEV

124/ old patient with gastritis low mcv (case of IDA ), what’s your management?
A. IM iron
B. Oral Iron
C. folate Something else
Answer: ​IM iron

125/ known case of depression came to ER after ingestion of bottle of drugs he is


comatose with dilated pupil what drug he ingested?
Answer: ​Amitriptyline

126/ lichen sclerosus cause which malignancy:


Answer: ​squamous-cell-carcinoma

127/ confusion, anemia, deranged kidney function, peripheral blood smear showed
schistocytes and fragmented RBCs, hemolytic panel was elevated, antibodies of which is
most likely implicated in the pathogenesis of the disease​​?
A. vWB
B. ADAMTS13
C. VIII
Answer: ​ADAMTS13
128/ Ice craving?
Answer: ​Pagophagia
129/ What is the third layer of the scalp?
Answer: ​Aponeurosis
130/ Fragile X syndrome feature:
Answer: ​Macroorchidism
131/ Women complains of the “worst headache of I’ve ever felt”. What could be the
cause?
A. Ruptured berry aneurysm
B. Pulmonary embolism Migraine
13

C. Cluster
Answer: ​A Ruptured berry aneurysm
132/ Trauma case with pancreatic injury and leakage into
A. Omentum Bursa
B. Subphrenic
C. Subhepatic
D. Gutter
Answer: ​A
133/ Longest phase of cell cycle?
Answer: ​Interphase
134/ Lysosomal trafficking regulator?
Answer: ​chediak higashi syndrome
Chédiak–Higashi syndrome is a rare autosomal recessive disorder that arises from a mutation
of a lysosomal trafficking regulator protein, which leads to a decrease in phagocytosis.
Usually the patient has blue eyes or albenism
135/ Diarrhea in a group of ppl after a wedding symptoms start 8-12 hrs ?
A. staph aureus
B. e coli
C. salmonella
Answer: ​A
136​​/ Scd with pallor low hb looks ill with urti then developed dyspnea ?
A. prbc transfusion
B. Analgesia and fluid
C. Iv antibiotics
Answer: ​B
137/ Leukemia on chemo came to er , pancytopenia and fever and myalgia ?
A. iv ceftriaxone
B. referral to onco
Answer : ​A
138/ Transverse presentation risk with?

A. Gynecoid pelvis
B. anthropoid pelvi​s
Answer: ​B

139/ Risk ratio


140/ Odd ratio
141/ Strange table with analytic method
Answer :

142/ Athlete with calf click and pain can't stand on tiptoe ?
A. calcaneal tendon
14

B. Rectus femoris
Answer: ​A
143/ Positive lachman test ..
A. cruciate anterior
B. Cruciate posterior
C. Lateral collateral
D. Medial collateral
Answer: ​A
144/ Howell jolly bodies found in
Answer: ​SCA
145/ Csf circulate in
A. subarachnoid
B. Epidural
C. Subdural
Answer: ​A
146/ Triangle bounded by hyoid bone and 2 bellies of digastric muscle ?

A. submandibular triangle
B. Submental
C. Posterior
Answer: ​B

147​​/ Infant with forceps delivery Injury to mastodostylo foramen


Wt sensation will be impaired.. ?
A. taste of ant 2/3 of tongue
B. Unable to blink with left eye
C. Loss of sensation in left face
Answer: ​A
148/ Rta with some Jugular foramen nerve injury Which muscle least affected with basal
skull fracture ..?
A. Sternocleidomastoid
B. Trapezius
C. Styloglossus
Answer: ​C
149/ Case of hyperaldosteronism ( HTN hypokalemia) how to diagnose,?
A. Acth supp test
B. Dexamethasone supp test
C. Aldosterone renin ratio
D. Serum catecholamine

Answer: ​C
Pheo >> 24 urine metanephrine
Cushing >> morning cortisol
15

Conn >> ald:renin ratio


150/ Which will support diagnosis ( case of hyperaldosteronism) ?
A. Vitiligo
B. Striae
C. Hyperpigmentation

Answer: ​C​ / ​check for polyuria


151/ Child on steroid came for vaccination has stria ?
A. Give all
B. Spare live attenuated until off steroid

Answer: ​B
152/ Triple-toxoid vaccine?
Answer: ​Dtap
153/ Enlarging uterus 18 weeks , amenorrhea for 12 weeks , raised bcg, small fetus?
Answer: ​Incomplete mole
Incomplete with fetus / large for GA
complete without fetus/ Snow storm and Empty uterus

154/ Neck mass , fna showed amyloid ?


A. Papillary
B. medullary
C. follicular
D. hodgkin lymphoma
Answer: ​B
155​​/ Child Case of malabsorption with low bicarb low k high creatinine weight on low
percentile Wt is pathophysiology?
A. Hydrogen excretion
B. Chloride absorption
Answer: ​A

DD are :
Cf its wrong as acidosis in cf its alkalosis

Barrter and gitelman its normal creatinin

Mostly this RTA type 1


Hypokalemia
Acidosis metabolic
Recurrent stone formation elevated RFT
It has defect in distal tubules in hydrogen exerection
156/ Child with diarrhea , same episode last month when he went on a trip ,, ?
A. Chronic giardia
16

B. Chronic salmonella

Answer: ​A
157/ what is the mode of inheritance of Galactosemia
answer : ​AR​​..
158/ 2 normal phonemically parents , chances to have normal child?
A. 25%
B. 50%
C. 75%
D. 100%
Answer: ​A
159/ Screening for cervical cancer ?
A. Every 3 yrs for women above 65 yrs
B. Every 6 yrs for women 30-65 yrs
Answer:
21-30 q3y pap
30-65 q5y HPV
160/ Man HTN on medication , smoker came with slightly high blood pressure reading
150/something Wt is secondary prevention ?
A. Smoking cessation
B. Hypertension treatment
C. Screening for colon cancer
Answer: ​C

151/ Old female on 4 anti HTN still hypertensive came with hypokalemia ?
A. Acth suppression test
B. Dexa suppression test
C. Aldosterone renin ratio
Answer : ​C
152/ Something about pheochromocytoma
Answer :​Lab catech in urine / Rx Alpha blocker/ SS Flushing Headache Htn

153/ Hearing voices from fridge , thinks food is poisoned ?


A. Delusion
B. auditory hallucination
Answer : ​B
154/ Most antipsychotic to cause involuntary movement ?
A. Risperidone
B. Haloperidol
C. Lorazepam
Answer : ​B
155​​/ Came with her father with tachypnea and tingling she failed a test?
17

A. Hyperventilation syndrome
B. Panic attack

Answer :​A
156/ Type of GN in goodpasture syndrome?
Answer : ​ nephritis with normal compliments / Rapidly progressive GN = Cresentic GN
157/ Type of change in DM nephropathy case with low GFR ?
A. Mesangial expansion
B. Thin glomeruli
C. Membranous mdri esh
Answer : ​A
158/ Nerve to supply posterior compartment of leg?
A. Tibia nv
B. Common fibular
C. Superficial fibular
Answer : ​A
159/ Injury to obturator nerve which ms injured ?
A. Quadriceps
B. Adductor longus
C. Adductor magnus
Answer : ​B
160/ IUD with watery discharge and abdominal pain ?
A. PID
B. Bacterial vaginosis

Answer : ​A
161/ Increasing dysmenorrhea associated with ?
A. Copper IUD
B. levonorgestrel IUD
Answer : ​A
162/( 15 yr old with few pubic hair and developed 2 ry sexual characteristic but she has
amenorrhea) wt is diagnosis
Answer :​Androgen insensitivity
163/ Cancer in pregnancy ?
A. Breast
B. cervical
C. uterine
Answer : ​A
164/ Stroke risk in female ?
A. Hypercoagulable state
B. Ventricular hypertrophy
C. Hormonal replacement therapy
Answer : ​C
18

165/ Case of lightheadedness , mother died at young age for cardiac event examination
normal , How to diagnose?
Answer :: ​Echo

166/ Convulsion in pt taking antidepressant and digoxin , ?


Answer : Toxicity digoxin?
167/ OCP ?
A. Estrogen and prevent ovulation
B. Progesterone and prevent ovulation
Answer : ​A
168/ Antibiotic for epididymo-orchitis?
A. Ceftriaxone
B. Doxycycline
Answer : ​A /Ceftriaxone IM once + azithromycin or doxycycline
169/ Mastitis case ?
A. Cephalexin
B. Ceftriaxone
Answer : ​A
170​​/ Child with barking cough wt is complication?
A. Epiglottitis
B. Meningitis

Answer : ​B as caused by parainfluenza virus which complicated by meningitis


171/ Pt with needle prick of hep C one year ago , wants to breastfeed wt is
contraindication?
Answer :: ​Cracked nipple
172/ Baby with jaundice mom rh -ve , Abo incompatibility?

A. Due to maternal antibodies causing agglutination of fetal


B. Fetal antibodies causing agglutination of maternal
C. Two other fetal and maternal i don't rmmbr
Answer : ​A
173/ Case of weight loss and irritability with bradycardia and mass in the neck?
A. Goiter nodular
B. Hyperthyroid
C. Hypothyroid
D. Thyroid ca
Answer : ​A
174/ Elder patient with respiratory distress CT angio show PE what is the most important
investigation options were :
A. V/q mismatch
B. CT spiral
C. Venography
19

Answer: ​B

175/ During morning symptoms relieved and during the day increased what is the path
Mg pathophysiology.
Answer :​ Anti ach antibodies

176/ Female with Depressed she found in comta and dilated pupil
A. SSRI
B. Burporin
C. TCA
Answer : ​C
177/ Increase protein in a pregnant woman what test we must to evaluate the function of
the kidney ?
Answer:​ ​Albumin/ creatinine ratio

178/ What is the most common lesion in perianal area above 50s.
A. Anal fissure
B. Hemorrhoids
Answer : ​B

179/ Pt can’t control his erection since 2 months, he woke up every morning with
erection, he had same condition 3 years ago, refer to whom?
A. Urology
B. Psychiatry
C. Neurology
D. Endocrine
Answer: ​B
180/ place to take liver biopsy?
A. Subclavian or subcostal
B. 3rd or 6th intercostal
C. 7th intercostal
D. 10th intercostal
Answer : ​C
181​​/ mechanism of action of metformin in muscle
A. Increase secretion of glucagon
B. Reduce glucose reuptake
C. Increase the gluconeogenesis
D. Increase fatty acid oxidation
Answer : ​D
182/ Someone consumed meat ( no time duration or symptoms ) ( incomplete Q)
Salmonella
20

Staph
Answer : depend about the case
If said undercooked its t.saginata
8-72 hours with salmonella
183/ 5 years (i think) what you'll give him for travelers diarrhea
A. Azithro
B. Cepro
Answer : ​A
184/ Which nerve cause “winged scapula”?
A. Axillary nerve
B. Long thoracic nerve
C. Inferior scapula nerve
Answer: ​B
185/ Pediatric pt with distal fracture of radius and ulnar what’s called this ?
Answer: ​ inward colls , outward smith
186/ fracture in neck of humerus which nerve will be affected?
Answer:​ axillary nerve / deltoid
187/ Fracture of shaft of humerus which nerve will be affected?
Answer: ​Radial nerve
188​​/ fracture in proximal tibia (with pic) what to do?
A. CT tibia
B. MRI femur
Answer: ​initial X-ray
189/ Gram -ve diplococci?
Answer: : ​Neisseria gonorrhoeae
190/ Gram -ve bacilli, oxidase -ve motile ?
Answer: ​Salmonella
191/ pt had radical mastectomy which will be affected?
A. Adduction
B. Abduction
Answer: ​A
192​​/ fracture of tibia which nerve will affected? ( wrong Q )
A. Superficial pudendal
B. Deep pudendal
C. Common pudendal
Answer: ​if In head : common
193/ Antidote of paracetamol
Answer: ​N-acetyl.
194/ Pt takes sildenafil what is contraindicated?
A. Aspirin
B. Beta-blocker
C. Nitrate
Answer: ​C
21

195/ Found in SCA


Answer: ​Howel jowell bodies
196/ Open fracture of forearm in pediatric pt what is the initial Tx?
A. Closed reduction with cast above elbow
B. Closed reduction with cast below elbow
C. Surgical department, ... & fixation
Answer: ​C
197/ cancer associated e polycystic ovary?
A. Ovary
B. Breast
C. Endometrial cancer
Answer: ​C
198/ When to do pap smear?
Answer: ​21-30 q3y pap ./ 30-65 q5y HPV
199/ Which one important risk for coronary artery disease?
A. LDL
B. Total cholesterol
C. HDL
D. Triglyceride
Answer: ​A
200/ pt diagnosed e cervical incompetence in ... wk what to do?
A. Do cervical ligation immediate at time of diagnosis
B. Do cervical ligation at 14 wk
Answer : ​B / cervical usual done in second trimester 16-18 w
201/ pic of eye with tree like dendritic cell. What is the most likely the diagnosis ?
Answer : ​HSV
202/ Antipsychotic drug causes decrease lipido?
Answer : ​ssri or tca
203/ loss of sensation in middle of palmer... which nerve affected?
Answer :​Median
204/ fishy discharge?
Answer :​BV
205/ Cancer comes along with pregnancy?
A. Breast
B. Ovary
Answer: ​A
206/ Pediatric e pubic hair no other signs of puberty?
A. Congenital adrenal hyperplasia
B. Premature adrenarche
Answer: ​B
207/ What i don’t remember it very well but it was something like this: pt e -3
osteoporosis, what is the Tx?
A. Vit. D
22

B. Estrogen
Answer: ​Bisphosphonate
208/ Fibroadenoma case: freely mobile + another Q what to do?
Answer : ​US
209​​/ Bad prognosis for schizophrenia?
A. Family Hx
B. In young
C. Acute
Answer :
210/ what is good prognosis for breast lump?
Answer: ​Aspirate clear fluid without refill
211/ 3 Qs of appendicitis, 2 Dx & 1 what to do?
Answer : ​Acute appendicitis ..appendectomy laparoscopic
Appendicular mass .. conservative
212/ Pt had abdominal blunt trauma and pancreatic enzymes comes out anteriorly where
it will it go first?
Answer: ​Omentum
213 / splenic artery ligation which one will be affected?
A. Lt arch of colon
B. Lt suprarenal
C. Lt kidney
Answer :​ Fundus of stomach
214/ pt pediatric parent said that she look like she can't breath what to do?
Answer :​ if asthma IV corticosteroid , if no asthma intubate

215/ Male patient complaining of mass on his neck, presenting with weight lose and
intolerance of heat, On examination his skin on the limps is warm and normal body
temperature On apical auscultation of his heart revealed BRADYCARDIA !
What's your diagnosis
A- nodule goiter
B- hyperthyroidism
C-hypothyroidism
D- thyroid carcinoma
Answer : ​A
216/ hymen perforation in abuse?
Answer: ​6
217​​/ Patient with Raynaud's phenomenon was referred to gastroenterologist for
something in esophaguses if I remember right question was what antibody is the cause
A- anticentromere
B - anti scl 70

answer: Anticentromere/ need more detials


23

218/ A case and it mentioned us show snow storm appearance


A- complete hydatid mole
B- incomplete hydatid mole
Answer: ​A
219/ A case with abdominal pain and amenorrhea symptoms were going toward ectopic
in the question the beta- hcg was in 2000 the doctor gave her methotrexate
But later she came back again and her bhcg now is 6000 what is the management
A- exploratory laparoscopy
b- methotrexate again
C - salpingectomy
D- salpingectomy
Answer: ​ A
220/ Another question with amenorrhea and abdominal pain on investigation there was
tubal pregnancy what is the cause
A - early disappearance of zona pellucida
B- persistence of zona pellucida
Answer : ​A
221​​/ Case about neonatal meningitis what is the most common organism
A- streptococcus agalactiae
B- listeria monocytogenes
C- neisseria meningitidis
Answer : ​most common Gbs is agalactea
222/ Question about pregnant say which is Indirect indicator for GBS infection
A- body temperature > 39
B- rupture of membrane > 18 hours
C- previous infant with GBS infection
D- pregnancy or (labor I don’t remember) < 38 weeks
Answer :​ C ( indirect factor )
A sure indication that infection already happened then treat
B its risky but not sure that it happen give prophylaxis
223/ Treatment of salmonella that is chloramphenicol resistant
a- another dose chloramphenicol
B- ciprofloxacin only
C- ciprofloxacin and ampicillin
Answer : ​B But make sure he’s an adult
224/ A case with ear discharge and the question mentioned involvement of mastoid what
is the treatment
A- amoxicillin
B- cefotaxime
Answer : ​B if no ceftriaxon
225​​/ A question of child came with seizure his parent said he never had seizure before ( I
think there was other information that made think of meningitis in the question) the
patient was given lorazepam for his seizure which controlled it what is next step?
24

A. Lumbar puncture
If blood culture there its first then Give antibiotic

226/ Child with bilateral swelling in front of ear and behind mandible what is most likely
complication
A- encephalitis
B- hearing loss
C - facial nerve paralysis
Answer : ​A
227/ Doctor was doing delivery I think by forceps by mistake he crushed content of
stylomastoid foramen which of the following will be present in baby
A- loss closure of eyelid
B- loss taste sensation from anterior 2/3 tongue
Answer: ​A
228/ A question in with there was chest x ray with right hilum is large the question is it
tell you cancer of the lungs in that position there was actually a red circle I think on chest
x Ray on the abnormality the question was lung cancers at that marked position cause
compression of vasculature which vessel most likely compressed
A- subcleaven
B - superior vena cava
Answer : ​B
229/ Which of the following pelvis shapes is associated with deep transverse arrest
A- gynecoid
B- Android
C- anthropoid
D- palatyploid
Answer : ​B
230/ Meckel diverticulum location
Answer : ​terminal ileum

231/ Best treatment for Gestational diabetes


A- insulin
B- metformin
Answer : ​ A
232/ Patient addict with pinpoint pupil respiratory depression antidote ?
Answer : ​naloxone

233/ Patient with hemorrhagic shock in this patient the activated baroreceptor will cause
decrease witch of the following
A- venous capacitance
B- coronary blood flow
C- heart rate
Answer : ​A
25

234/ Diabetic complaining of incontinence which study will aid confirming diagnosis
A- urodynamic study
B- residual urine
Answer: ​A / Residual is initial test
235/ Female patient didn’t have a known disease in the case presented with urgency ,
frequency and dyuria physical examination showed suprapupic tenderness what is the
cause there was not cough
A- stress
B- hypertonic
C- hypotonic
D- sensory irritative
Answer : ​B / urgency symptoms
236/ Long case about dyslipidemia the patient was given antilipidemic and he had facial
flush his doctor divided his aspirin dose to 3 times a day which improved it what is
antilipidemic described
Answer: ​niacin

237/ question about tetralogy of Fallot which is a component


Answer : ​pulmonary stenosis

238/ Female with amenorrhea secondary sex characteristic normal but absent uterus
Answer: ​mullerian agenesis

239/ 16 year old or something close anyway complain of irregulars Menses every 2
months and she had her first menses 6 months ago what is management
Answer: ​reassurance

240/ A patient with broken hair and he admit of pulling hair when he is under stress
diagnosis?
Answer: ​trichotillomania

241/ 6 months child which of the following gross motor milestone he can have
A- sit unsupported
B- stand holding furniture
C- roll from prone to supine
D- roll from supine to back
Answer : ​A

242/ Mother brought her 3 years old to see his language development he can
communicate easily something like this and 75% of his speech is understandable he ask
questions frequently there was 2 others I forgot
A- developmental expressive aphasia
B-something dysfluency I think speech
26

C- normal development for age


Answer : ​A
243/ Absent red reflex which condition
Answer: ​retinoblastoma

244/ Tumor in somewhere in head compressing 3rd part of maxillary artery which branch
is affected
A- sphenopalatine
B- middle meningeal
C- infra alveolar artery
Answer : ​A

245/ Tumor in cavernous sinus patient have diplopia and blurred vision which cranial
nerve
Answer: ​abducent

246/ Depression cause loss of pleasure and so on decrease in what substance cause this
Answer: - ​serotonin

247/ Which is given with analgesics to reduce their side effect


Answer: ​metoclopramide

248/ Patient was given drug next day he came with twisting movement what drug was
given
Answer: ​metoclopramide

249/ Patient with malaria taking antimalarial I don’t remember which had symptoms they
were of anemia what enzyme is the cause
Answer: ​glucoses 6 phosphate dehydrogenase

250/ Case about patient for check up and which lab is strongest risk for heart disease
A- ldl
B- hdl
C- total cholesterol
D- tag
Answer: LDL
251​​/ Which type of exercise help maintain bone mass while aging ?
Answer : ​Weight bearing Or walking/ Isotonic (Aerobic )
252/ Sulfa drug reaction
Answer: ​blister and velocous hue

253/ Upper arm trauma loss of addiction of digit nerve injury


A- ulnar
27

B- median
Answer: ​A
254/ 14 month old child with grade 2 murmur and 2 cm ventricular septal defect how to
manage?
A- surfer
B - watchful waiting
Answer: ​B
255/ Dog bite ?
Answer: ​polymicrobial

256/ Case with heart symptoms on auscultation ejection systolic murmur heard best at
right upper sternum
A- calcified aortic stenosis
B- mitral regurge
Answer: ​A
257/ Patient with right shoulder pain his heat is normal but he have pleurisy which part of
pleura transmit pain
Answer: ​mediastinal

258/ Main component of air blood barrier


A- pneumocyte I
B- pneumocyte II
Answer : ​A
Note: type 2 secretes surfactant

259/ Painless ulcer in penis healed then rash appeared at soles and palms diagnosis
Answer : ​syphilis

260/ Patient with symptoms there was erythema modicum and x Ray showed bilateral
lung infiltrate with hilar lymph node.
Answer :​sarcoidosis

261/ Which of the following is indication for tonsillectomy


A- asymptotic enlarged tonsils
B- bacterial pharyngitis or tonsillitis
C- sleep apnea syndrome
Answer : ​C
262/ Radical mastectomy winging of scapula
Answer :​long thoracic nerve

263/ Ameoba prevention


A- hand washing
B- don’t eat undercooked vegetables
28

Answer :​ B ​There was no boiling

263/ Mers prevention


Answer :​ ​hand wash

264/ Patient who exercise he have osteoarthritis on examination doctor heard criptition in
right knee there was fluid what to next
A- x Ray both knees and treat with ibuprofen
B- aspirate the right knee fluid
Answer : ​A
265/ 16 years old with recurrent swelling of lips caused by
Answer :​c1 Esterase inhibitor

266/ Question of patient was diagnosed with schizophrenia but he had symptoms for 1
month which of the following against schizophrenia diagnosis
Answer :​duration of symptoms

267/ Woman in the ER she was staring at nothing and saying you can’t kill me what is
this symptom
Answer :​hallucination

268/ Someone saying his thoughts are being broadcasted to others through radio
Answer :​thought broadcasting

269/ Psychiatry question patient feels swatched from self and world not real
Answer :​derealisation

270/ Patient with scrotal swelling appeared after lifting heavy object and there’s
abdominal pain
What to do
A- refer to urologist
B- refer to surgery
Answer : ​B
271/ Patient with crushing retrosternal chest pain something like that it meant to me very
severe pain other findings it looked to me MI
Answer : ​have a lot of DDX

272/ Which of the following most likely acute ST elevation MI


Answer ​: ongoing chest pain with .3 mm st elevation in leads II , III and AVF

273/ Patient with ischemia to the heart with Brady cardiac and something else the doctor
confirmed ischemia to SA node and AV node which artery the lesion in
A- right coronary artery
29

B- left coronary artery


Answer: ​A
274/ A patient 5 weeks post MI came with chest pain and other symptoms that is of MI
labs show troponin normal diagnosis
A- recurrent MI
B- dressler syndrome
Answer: ​B
275/ Patient ask about driving post Myocardial infarction what to tell him
A- driving increase risk of recurrence
B- he can drive after 4 weeks
Answer: ​B
276/ Patient came with unilateral knee pain and urethral discharge for 1 month his cults
for urine is negative what is the cause
A- Reiter syndrome
b- septic arthritis
Answer: ​A
277/ Reactive arthritis or retire syndrome
Answer: ​conjunctivitis, urethritis, and arthritis
occurring after an infection, particularly those in the urogenital or gastrointestinal tract
278/ Risk of colorectal cancer recurrence is strongly related to :
a) Age at diagnosis
b) Stage
c) Family history
Answer: ​B
279​​/ Which of the following method is rapid and best for complete gastric evacuation?
a) G lavage
b) Manual induce Vomiting
c) Syrup
d) Active charcoal
Answer: ​D

280/Which one of these patients with pneumonia will you treat as outdoor patient:
a) 80 Years old with 104 F temperature, BR 24/min PR 126/min, BP 180/110
b) 60 years old with 102 F temperature BR 22/min PR 124/min, BP 160/110
c) 50 years old with 98 F temperature, BR 20/min. HR 110/min, BP 180/110
d) 80 years old with 96 F temperature, BR 18/min, HR 70/min, BP 110/80
Answer: ​C​ ​according to Curb65
281/ Old patient stopped smoking 10 years ago, suffering from shortness of breath after
exercise but no cough and there was a table FEV1=71% FVC=61% FEV1/FVC=95%
TLC=58% What's the dx?
a) Restrictive lung disease
b) Asthma
c) Bronchitis
30

d) Emphysema
e) Obstructive with restrictive
Answer:​ A
FEV/ FVC is normal so its restrictive
In obstructive or mixed its below 70

282/ A case and it mentioned us show snow storm appearance


A- complete hydatid mole
B- incomplete hydatid mole
Answer:
283/ A case with abdominal pain and amenorrhea symptoms were going toward ectopic
in the question the bhcg was in 2000 the doctor gave her methotrexate
But later she came back again and her bhcg now is 6000 what is the management
A- exploratory laparoscopy
b- methotrexate again
C - salpingectomy
D- salpingectomy
Answer: ​A
284/ Another question with amenorrhea and abdominal pain on investigation there was
tubal pregnancy what is the cause
A - early disappearance of zona pellucida
B- persistence of zona pellucida
Answer: ​A
285/ Case about neonatal meningitis what is the most common organism
A- streptococcus agalactiae
B- listeria monocytogenes
C- neisseria meningitidis
Answer:
286​​/ Question about pregnant say which is Indirect indicator for GBS infection
A- body temperature > 39
B- rupture of membrane > 18 hours
C- previous infant with GBS infection
D- pregnancy or (labor I don’t remember) < 38 weeks
Answer: ​all Correct Ref: up to date / B
287/ Treatment of salmonella that is chloramphenicol resistant
a- another dose chloramphenicol
B- ciprofloxacin only
C- ciprofloxacin and ampicillin
Answer: ​B/ in adult
288​​/ A case with ear discharge and the question mentioned involvement of mastoid what
is the treatment
A- amoxicillin
B- cefotaxime
31

Answer: ​B​ ​Note : no vancomycin or ceftriaxone ( need Ref)

289/ Child with bilateral swelling in front of ear and behind mandible what is most likely
complication
A- encephalitis
B- hearing loss
C - facial nerve paralysis
Answer : ​A

290/ Doctor was doing delivery I think by forceps by mistake he crushed content of
stylomastoid foramen which of the following will be present in baby
A- loss closure of eyelid
B- loss taste sensation from anterior 2/3 tongue
Answer :​A
298/ Female with amenorrhea secondary sex characteristic normal but absent uterus
Answer: mullerian agenesis
291​​/ 6 months child which of the following gross motor milestone he can have
A- sit unsupported
B- stand holding furniture
C- roll from prone to supine
D- roll from supine to back
Answer : ​A
292​​/ Mother brought her 3 years old to see his language development he can
communicate easily something like this and 75% of his speech is understandable he ask
questions frequently there was 2 others I forgot
A- developmental expressive aphasia
B-something dysfluency I think speech
C- normal development for age
Answer : ​C

293​​/ Patient with crushing retrosternal chest pain something like that it ment to me very
severe pain other findings it looked to me MI
Mechanism of action of analgesic use
A- inhibition of cyclooxygenase
B - agonist of CNS Mu receptors
Answer : ​B

294​​/ what is the first enzyme elevated in Acute liver disease ?


Answer : ​AST
295​​/ Pregnant healthy 12 w spleen palpable gum bleeding ,
A. Platelet 15
B. Itp
C. Gestational thrombo
32

D. Thromboembolic
Answer : ​D if platelet count less than 20.000 ITP / more than 20,000 Thromboembolic
296​​/ a case of ITP platelet was 15k what is the next step in management ?

A. IV gamma globulin
B. Platelet transfusion
C. Splenectomy
Answer : ​corticosteroids
First line : steroid / In children its IVIG
Second line : IVIG
Third line : Splenectomy

297​​/ a scrino was given. Lab showed high IgG , low IgA , low IgE, High Ca, what is the
most likely dx?
Answer: ​Multiple myeloma / If ca normal its MGUS (monoclonal gammopathy of undetermined
significance )
CARB
Multiple myeloma:
Ca high
Renal failure
Anemia
Bone pain
298​​/ Principle in urge incontinence management ?
A. Bladder training and physiotherapy
B. Medical
Answer : ​A
299​​/ pt presented with flu like symptoms and jaundice anorexia and normal abdominal
examination in the lab: high bilirubin, ALT and AST mild elevation and high ALP
‫ ﺟﺎﻧﻲ ﺳﻮال ﻏﺮﯾﺐ ﻓﯿﻪ ﻗﻠﻲ واﺣﺪ ﺟﺎه‬flu like and jaundice anorexia normal abd examination
A. Gilbert
B. Stone
C. Hep A
D. Primary biliary cirrhosis
Answer : ​A
300/ HIV medication that causes bone marrow Suppression
Answer : ​Zidovudin = Retrovir + tenofovir
301/ Young age with uric acid stone treated surgically , he has normal serum uric acid ,
how to prevent recurrence ?
A. Alopurinol
B. Sodium citrate
Answer : ​B
302/Genetic study for gene responsible for lung ca metastasis to brain
33

A. Gftr translocation
B. Gftr mutation with ALK mutation
C. BAV V600
D. No need to do at this stage
Answer : ​B
303/ / Mechanism of action of analgesic use
A- inhibition of cyclooxygenase
B - agonist of CNS Mu receptors
Answer : ​A

304/ In late deceleration , 90% effacement , what to do !


A. - SVD
B. - CS
C. - vacuum
D. - forceps
Answer: ​B
305/ Energy of the cell
A. - nucleus
B. - nucleulus
C. - cytoplasm
Answer : ​which is cytoplasm reticulum /
306/ Transfer inside and outside the neucleas ?
Answer​: ​Cytoplasmic gtp

307/ Hyponatremia + high urine osmolarity


A. - diabetes insipidus
B. - SIADH
Answer: ​B
308/ Low urine osmolarity
A. -​​ insipidus
B. - SIADH
Answer : ​A
309/ Rice dependent + fatigue
A. - vit A
B. -vit C
C. - B1
D. - B3
Answer : ​C
310/ Niacin SE ?
Answer : flushing of the face + mild dizziness; warmth, redness, or tingly feeling under your
skin; itching, dry skin; sweating or chills; nausea, diarrhea, belching, gas; muscle pain, leg
cramps; or. sleep problems (insomnia). + ​blurred vision , cystoid macular edema and toxic
amblyopia
34

311/ HTN pt , newly DM , which hypoglycemic you will give ?


Answer: ​Metformin
312/ bleeding in napkin only while whipping in the bathroom, us normal , pelvic scope
normal , where is the lesion ?
A. - cervix
B. - vulva
C. - urethra
D. - pelvic
Answer: ​A
313​​/ Appendicular mass case , then ask me about ttt of acute appendicitis ????
A. - Sx
B. - conservative
Answer: ​ B if acute surgery
314/ GERD Sx + peptic ulcer for 1 month , but while palpation there is epigastric mass ,
what to do next
A. - ppi for 6 wks
B. - h pylori
C. - us
Answer: ​C
315/ Normal thing you suspect among all elderly for there cognitive impairment , what to
suspect in ur test ?
A. - low IQ
B. - hearing loss related to age
C. - c test
Answer: ​B
316​​/ DM1 case , confirmatory test
A. - DR3
B. - urine dipstick
Answer:​ B
317/ Nephrotic case , what to support yr Dx
A. - previous impetigo
B. - autoimmune disease
Answer: ​A
318/ Most common pancreatic cancer
Answer: ​Adenocarcinoma

319/ 14 years old with microcytic hypochromic anemia with normal ferritin and normal Hb
A2 what to do next:
A. Iron level and TIBC
B. Genetic testing
Answer: ​A
320/ Post MI autopsy what will u find in the brain:
35

A. Red neurons in hippocampus


B. Infarction in areas supplied by left MCA
Answer: ​A

321/ Post splenectomy with splenic artery ligation which will be affected:
A. Left colonic flexure
B. Gastric fundus
Answer: ​B
322/ In SCA
A. One subunit is replaced
B. 3 subunit deleted
C. Mutation.....
Answer: ​ A

323/ Left knee swelling and pain synovial fluid analysis: turbid WBC 15000 with PMNs
85% ( they did not put culture or Gram stain or crystals):
A. -Septic arthritis
B. -Gout
C. -Pseudogout
Answer: ​C
324/Pregnant in labor received fentanyl now fetal heart rate dropped and she is taken to
OR or delivery room In left lateral position and feel light headed what to do next
A. Ask for forceps
B. Some position
C. Fentanyl
Answer:​​ ​Fentanyl induced hypotension / Treatment ephidrin

325/ Pt with pain abd. Pain and fornix tenderness received ceftriaxone how ever she did
not improve, what is the organism:
Answer:: ​C. Trachomatis/ Add doxycyline

326/ Child with meningitis LP culture revealed G +ve cocci in pairs: what is the antibiotic
combination:
A. Vancomycin + ceftriaxone
B. Penicillin + vancomycin

Answer: ​A
327/ Pregnant 41 week came for induction of labour. She has some mucoid blood tinged
discharge from vagina and cervix dilated what is the next step:
A. Artificial rupture of membranes
B. Oxytocin
C. Catheter
Answer: ​ incomplete Q
36

If cervix less than 3cm could use catheter balloon for dilation
If more than 4 cm favourable cervix do ARM
328/ About Valvular cancer what is true:
A. (Lesion description) in labia majora
B. Same description in clitoris
Answer: ​A
329/ Take iron pill came to Er with Iron over
load , what initial to do ??

A.Gastric lavage
B.Activated Charcoal
C. Solution( forget name)
Answer: ​phlebotomy as primary and deferoxamine as secondary

330/ . Regarding iucd ;

A.-Changed every 6 months


B.-cause perforation
C.- cause bleeding
Answer: ​C
1.expulsion 2. PID 3. Bleeding (only available option)
331​​/ Primigravida at 2 stage was given fentanyl in last scenario mentioned fetal heart
drop and mother feel dizzy and lightheadedness , what to do ??

A.Give another dose of fentanyl


B.Cal nurse for forcep
C.Put her in fowler's position
D.No GA in option
Answer: ​lateral position . if lateral then IV fluid and IV ephedrine
332/ There was scenario about person came from africa and has symptoms like fever ,
abdominal pain ....

A.yellow fever
B.Ebola
C.Dengue fever
D.No malaria in option
Answer: ​A ( but incomplete Q )
333/Drug contraindicated in closed angle glaucoma ;

A.No brimonidine in choice


B.No atropine in choice
Answer: ​ACE
A: atropin C: cimetidin E: epinephrine / dipivefrine
37

334/about dorsalis pedis Will dr, palpate the dorsalis of the foot which is lateral to
tendon?

A.Extensor hallucis longus


B.Extensor hallucis digitorum
Answer: ​A
335/ complication of croup
A. Pneumonia
B, epiglottis
Answer: ​A
336/ complication of bronchiolitis ?
A. Pneumonia
B, epiglottis
Answer: ​ A
337/ ttt of severe acne
A. .Isotretinoin
B. .Abx
Answer: ​A
338/ cell secrete melatonin or melanin
Answer: ​Melanocyte
339​​/ Turner what percentage of second baby with turner
Answer: ​30%
340/ Also turner , what complication or imp disease will have ;
from choices there was
A.cardiac ,
B.ovary disease
Answer: ​A

341/ Still birth which week ?

A.20
B.22
C.24
D.26
Answer: ​C
342/ Rib fracture what proper test ;

A. Anterior - posterior chest x-ray


B. .Posterior - anterior x-ray
C. .Oblique x-ray
D. .Ct scan
Answer: ​C
38

343/ Pt on phenytoin for long time , complain of hair growth , her all test is okay , what to
do

A.Continue on phenytoin
B.Replace it to another medication
C.Stop phenytoin
Answer:​ C / need more info but Generally Stop and duration is important
344 , what anticoagulant given to pregnant

A.LMWH
B.HEPARIN
C.WARFARIN
D.Unfractured heparin
Answer: ​A /then at week 36 shift to unfractionated heparin
345/ ; tt , group A or B streptococci ( forget )
Answer: ​Penicillin If allergy Clindamycin
346/ , which drug prevent clot formation
A.Clopidogrel
B.Heparin
C.Streptokinase
Answer: ​ A/ prevent
347/ q about runner complain pain
Answer:​ Incomplete senario
348/ Hemophilia is X linked ?
answer: ​X linked rec 50% chance of males being affected if mother is heterozygous. No male to
male transmission/ Pic added / for the daughter 50% carrier

349/ other q in medial lower leg like this


A.Plantar fasciitis
B.Achilles tendon rupture

Answer: i​ncomplete but pic added for DDX


350/ Study selecting each 10th family?
Answer ​Systematic Random
351/MERS:
Answer: ​66:1
352/ Lung Cancer, asking about risk associated:
Answer: ​70%
353/ Bladder Cancer:
Answer: ​ 2.25
354/ Prostate cancer and fat diet:
Answer: ​OR 5.2
355/ Typhoid:
39

Answer: ​100
356/ Expected child height:
Answer: [​Boy: (Father+Mother)/2+13] if [Girl: (Father+Mother)/2–13]
357 Relative Risk (RR):
Answer: ​ a/(a+b) c/(c+d)
358​​ Which is directly associated with smoking:
A. SCC/SSC of lung
B. otherwise Bladder Cancer
Answer: ​A
359. Most common nerve injury in humerus fracture:
Answer: ​: Radial —�> in shaft Axillary —�> in neck
360/ Trauma to pancreas, place fluid will collect:
Answer: ​omental bursa
361/ Thalassemia that causes stillbirth (Hydrofetalis):
Answer:​ Major a-Thalassemia (deletion in 4a, and normal 2b)
362/ Injury to posterior vagal trunk:
Answer: ​Esophagus
363/ Aortic coarctation most commonly associated with:
Answer: ​Turner Syndrome [Turner also has hypothyroidism]
364/17 Years old with breast lump, rubbery:
Answer:​​ ​Fibroadenoma​ (always small, mobile, painless, not related to cycle). If related to cycle
think about fibrocystic chain.
365/ Cancer associated with depression:
Answer: ​Pancreatic cancer.
366/ Longest stage of cell:
Answer : ​Interphase
367/ Shortest stage of cell:
Answer: ​Anaphase (Chromatin separation)
368/ Milestones: Your reference should be Nelson.
369/ Contraindication for circumcision:
Answer: ​Hypospadias​.
370/ Most common congenital anomaly in male genitalia:
Answer: ​Cryptorchidism​.
371/ Sulfa drug induced allergy:
Answer: ​Blisters​.
372/ Fixed drug eruption:
Answer: ​Plaques and patches.
373/ Organism associated with chronic Granulomatous:
answer: ​Staph. aureus.
374/ Chronic Granulomatous disease ?
Answer: ​NAPDH Oxidase deficiency.
375/ PCOS:
Answer: ​Increase risk for endometrial cancer.
40

376/ Lateral elbow trauma (or recurrent constructor injury):


Answer: ​Lateral epichondylitis [Tennis, Squash players]
377/ Golf players’ common trauma:
Answer:​​ Medial epichondylitis
* [All of the above ttx: Reassurance, rest and NSAIDs]
378/ Constipation, weight loss, OE there is pelvic mass, usually old age:
Answer: ​Colorectal cancer, needs colonoscopy.
379/ Mertformin action on muscle:
Answer:​ Increase glucose uptake.
380/ Dog bite is usually
Answer: ​Polymicrobial (or Pasturella).
381/ Cat bite:
Answer:​​ Pasturella malticeda
382/ TOF: 4 FEATURES.
383/ Patient with hemorrhage?
Answer: ​Decreased venous capacitance (through Baroreceptors)
384/: Pain in heel when waking up, decreased by walking dx?
Answer : ​Plantar fasciitis
385/ Hep. A vaccine missed by one week:
Answer:​ Give now.
386/ Meckel’s Diverticulum site:
Answer: ​Terminal/lower ileum.
387/ Pudendal nerve block in vaginal delivery, which organ is not affected:
Answer: ​Rectum.
388/ Pregnant with recurrent abortions and D/C came with bleeding in 1st trimester:
answer: ​Asherman Syndrome. If there is no Hx of D/C, it is most likely chromosomal.
389/ Cervical incompetence abortion is usually in
Answer : ​2nd trimester.
390/ Patient with IUD complaining of little secretion and abdominal pain:
Answer: ​Normal. But if there is fever → PID.
391/Female with multiple abortions (especially with Hx of SLE, DVT or any thrombosis
disease) —>
Answer : ​Antiphospholipid Syndrome
392/ Retinoblastoma (white reflex in pupil, or absence of reflex) treatment ?
​answer : ​Chemotherapy for reduction.
393/ Elderly with Alzheimer’s presented with bizarre behavior or change in level of
consciousness:
Answer ​Add Resperidone.
394/ Teenager student got bad grade on exam:
Answer : ​Justifying Rationalization​.
Talking about facts —> Intellectualization.
Helping others —> Sublimation.
Blaming others —> Projection.
41

395/ Patient in office suddenly got out:


Answer: ​Acting out.
396/ Side effects of atropine:
Answer. : ​Dry mouth, blurred vision.
397/ Unconscious patient with empty bottle and dilated pupils came to ER: Dilated pupils,
increased HR and other sympathetic symptoms especially if on antidepressant, the drug
used:
Answer :​TCA —> Amytriptaline
398/ Depressed patient on therapy found unconscious with empty bottle but has
metabolic acidosis, the drug used:
Answer: ​ Aspirin overdose​.
399/ Unilateral throbbing headache, photophobia, aura sometimes:
Answer: ​Migraine.
400/ Patient with breast mass, which lymph node to examine first:
Answer: ​Anterior axillary (pectoral).
401/ Dizygotic twins, what does that mean:
Answer: ​Di-Di Zygotic (Di-Di Chorionic) regardless of sex.
402/ Mechanism of emergency OCP:
answer: ​Prevent or delay ovulation.
403​​/ Mechanism of emergency IUD:
Answer: ​Prevent fertilization .
404/ Most sensitive age to steroids:
Answer: ​24 months — 96 months (2-8 Years).
405. Arteriosclerosis most risk factor:
Answer ​:Oxidized LDL.
406/ Most common side effect of anti malaria medication:
Answer: ​Itching (or skin pigmentation).
407/ Gene in colorectal cancer:
Answer: ​APC Gene (Chromosome 5). 5q22.2
408/ Sensation in anterior ⅔ of tongue:
Answer: ​Trigeminal nerve (CN5) Lingual nerve.
409/ Taste in anterior ⅔ of tongue:
Answer: ​Facial nerve (CN7) —> Chorda Tympani
410/ Taste & sensation in posterior ⅓ of tongue:
Answer: ​Glossopharyngeal nerve.
411/ Definition of Recurrent Abortions:
answer: ​3 or more consecutive abortions.
412/ Thrombolytic agent?
Answer :​Streptokinase / Alteplese
413/ Hormone recommended in infants:
Answer: ​Vitamin D.
414/ Primary aldosteronism associated with:
Answer: ​High Na, decreased K and hypertension.
42

415/ Incretin MOA:


Answer: ​Increases insulin secretion from pancreas.
416/ Metformin MOA: 4 major MOAs?
Answer: ​Increase glucose uptake in peripheral tissues, increase cell sensitivity to insulin,
decrease hepatic gluconeogenesis, decrease glucose uptake from GIT (+ effect on muscle).

A. Increase glucogenesis
B. Decrease peripheral uptake
C. Increases glycogenolysis
D. Enahance FA oxidation
Answer : ​D
417/ Hyperglycemia inhibits wound healing by:
Answer: (​impaired​ ​phagocytosis).
418/ Which drug causes angioedema:
Answer: ​ACEI.
419/ Site the surgeon will look for diverticulum?
Answer: ​Terminal/lower ileum.
420/ Premature menopause:
Answer: ​Before the age of 40.
421/ Age of Intussusception:
Answer: ​6 months to 36 months (up to 3 years).
422/ Thalassemia mode of inheritance ?
Answer:​ Autosomal Recessive.
423/ Percentage of Autosomal Recessive:
Answer: ​25% diseased/normal, 50% carriers.
424/Research about heart disease in two communities, we found that HD is more in A
community. What is the most important Q to ask?:
Answer: ​Age Group.
425/ Case-Control ?
Answer: ​Recall bias,
426/ Cross-sectional ?
Answer: ​Non-response bias.
427/ Meta-analysis ?
Answer ​:Publication bias.
428/ Lesser NSAID to cause GI symptoms:
Answer: ​Coliximib
429/ Hypertensive patient with gout, caused by:
Answer: ​Thiazide diuretics.
430/ Non-inflammatory acne ttx ?
Answer: ​Retinoic acid.
431/ Inflammatory acne ttx ?
Answer: ​Antibiotic (mild-moderate), Isotretinoin (severe-cystic-lesions).
432/ Palpitation with normal ECG ttx ?
43

Answer: ​Propanolol (b-Blockers)

433​​/ Pt C/O of vaginal bleeding .. O/E oral & vaginal ulcers and lymphadenopathy
Normal WBC
-syphilis
-chancroid
-HSV
-lymphogranuloma venereum
Answer : ​HSV
434/ hashimoto disease associated with cancer
Answer: ​papillary

435/ TTT of metoclopramide toxicity


Answer : ​diphenhydramine, benztropine

436​​/ In which disease we used combination of heparin and FFP


Answer: ​DIC

437​​/ prolonged BT and normal PT and PTT


Answer : ​Low platelets / VWD has prolonged PTT could be Bernard too

438/- bence Jones protein ?


Answer : ​Multiple myeloma

439/- Loss of sensation of 2/3 anterior tongue


Answer : ​CN: V trigeminal

440/- Loss of taste of 2/3 anterior tongue + mouth deviation


Answer :​ CN VII facial nerve

441/-diapers rash what is the TTT


Answer :​ Frequent changes of diaper + antifungel

442/-prophylaxis for meningococcal meningitis .. no rifampin in the choices


Answer : ​Cipro for adult / Ceftriaxon for children

443/ - Goblet cells ?


A. -stomach
B. -illium
C. - duodenum
D. - as. Colon
Answer : ​D
44

444​​/ Breast pain under riola and outer no mass ?


A. -ductile
B. -fibrocyst
C. -carcinoma
Answer:
445/ Temporal region injury what will affect ?
A. MCA
B. MMA
Answer: ​B
446​​/ Pt head injury he can’t bring spon by himself which lobe is affect ?
A. Temporal lobe
B. Occipital lobe
C. Parietal lobe
Answer: ​Cerebellum if not then check from temporal
447/ Child with tea urine , fatigue , abnormal movement , parents note skin eruption for (
unknown number of days) ?
A. -PSGN
B. -HUS
C. -IPT
D. HSP
Answer: ​TTP

448/ Breast pain from under areola till outer lateral ?


Fibrocystic disease
Ductal ectasia
Carcinoma
Answer: ​B
449​​/ Antiplatelet which decrease stroke mortality with have evidence decrease from 8%
to 4% in 10 years ? What number we need to prevent to treat stroke ? ‫ﻣﺎﻓﻬﻤﺘﻪ و ﺑﺎﻟﻀﺒﻂ ﻛﺬا ﻛﺎن‬
A. 4
B. 5
C. 10
D. 25
Answer: ​NNT maybe /8-4 =4 / 1÷4 =0.25 ×100 = 25

450​​/ Rash with non special virus symptoms only !


A. Infection mononucleosis
B. EBV
Answer:
451/ Child with Jaundice association with fever and vomiting disappear after that !
Answer:
45

452/ In ER .. One hour pt with MI ?


A. Tropenin
B. CK
C. AST
D. Lactate dehydrogenase
Answer: ​B
453/ ​A patient with pain in the hip increases with the up and down of the stairs and decreases
when walking in the flat area what is DX ?
Answer: C​hondromalacia also Called patellofemoral syndrome

454​​/ Pregnant lady in her 14th week of gestation came with complaint of nausea and
vomiting post meals. She also lost 15 Kg last week. Upon examination she was
dehydrated. What will you find in her labs?
a. Low bhcg
b. High progesterone
c. High TSH
d. H pylori

Answer:

455/ Pt with regular cycle without any complains they found cyst or mass over the Rt
ovary ?
A. -Corpus luteal
B. -theca luteum
C. -Follicle cyst

Answer: ​C

456 Polycystic ovary syndrome case… what’s the diagnosis ?


Answer: is ​Stein–Leventhal syndrome
457/​​ pt with mild anemia, what is the a thalassemia genotype ?
A. a-/a-
B. aa/- -
Answer: ​Both are right.
458/​​ ster bovis and colon cancer ​The question is what is the organism associated with colon cancer
Answer: S​treptococcus bovis (S. bovis) bacteria are associated with colorectal cancer and adenoma.

459/ ​ high energy in cell ?


answeR :Cytoplasmic GTP
460/ ​gilbert genotype
Answer: ​UGT1A1 & HUGBR1 /Chromosome 2 q37
461/ ​sever bronchiolitis management R respiratory rate 50 with tachycardia O2 90% ?
A. inhaled corticosteroid
46

B. ventilatory support
C. supportive care and hydration
Answer: ​B
462/ ​Psychic pt swallow safety pins now in small intestine with no air
A. discharge the pt
B. admit for observation and x-ray
Answer: ​B
463/ ​dumping syndrome
Answer: ​weakness, abdominal discomfort, and sometimes abnormally rapid bowel evacuation,
occurring after meals in some patients who have undergone gastric surgery. /dumping” or postprandial
hypoglycemia
464/ ​ meigs syndrome type of ovarian cancer
Answer: ​Sex cord stromal cells(fibroma)
465/ ​common brain tumor in adult
Answer: ​Glioblastoma multiforme / In children : Astrocytomas
466/ ​ ​ovulation date 28 days
Answer: ​ ( Cycle length -14)
467/ ​ ​copd mechanism
A. increase lung compliance,
B. decrease lung compliance
Answer: ​A
Common causes of decreased lung compliance are pulmonary fibrosis, pneumonia and pulmonary
edema. In an obstructive lung disease, airway obstruction causes an increase in resistance.
468/ ​lung volumes in pregnancy
Answer: T​idal volume only increases , the rest decrease
469/ ​ description of Drug eruption?
Answer : ​ulcer on penis and blister
470/ ​ X-ray ( SCAFE)
471/ ​X-rays( perthe)
472/ ​description of AF
Answer : ​HTN pt with irregular irregular rhythm and tachycardia
473/ ​ Type of cell in lung squamous cell carcinoma
Answer: ​Clara
474/ ​Gas gangrene after farm injury
Answer : ​c. perfringens
475/ ​Treatment of typhoid in 5 year old child
Answer: c​eftriaxone
476/ ​vaginal candidiasis treatment (by dose and route )
Answer: 1​50 mg as a single oral dose./ pregnant go for topical.
Ref :https://www.rxlist.com/diflucan-drug.htm#dosage
477/ ​Migraine prophylaxis in asthmatic
Answer: (​amitriptyline)
478/ ​ treatment of open angle glaucoma in COPD
Answer : ​oral acetazolamide
479​​/ ​Eclampsia pt on Mgsulfat for emergency C/S , after hour become tachycardic tachypneic B/P
110/70 .. Most likely reason for this ?
A. Mg toxicity
B. Hemorrhage
47

C. anesthesia induced hypotension


Answer: ​ A or B not supporting dx of Mg toxicity but Q is missing.
480/ ​patient with DM and HTN controlled on ACEI , furosemide and spironolactone ,electrolyte
balance is normal, what you should do:
A- stop furosemide
B- stop spironolactone
C- add digoxin
Answer: ​B
481​​/ ​Pt diagnose with anterolateral MI, PCI not available what’s the initial management:
A. Aspirin, streptokinase, BB, nitroglycerin
B. Aspirin, BB, nitroglycerin
C. Aspirin,BB,nitroglycerin, heparin
D. Aspirin,BB,streptokinase, heparin
Answer:
482/ ​Hand foot mouth syndrome
Answer:​ Coxascki virus
483/ ​Metoclopramide toxicity what to give ?
Answer:​ diphenhydramine

484/​​. You are reading a population study that states that 90% of lung cancer patient are smokers
while 30% of lung cancer patient are non-smokers. What is the specificity of using smoking as a
predictor of lung cancer?
a) 10%
b) 40%
c) 30%
d) 70%
e) 90%
Answer ​: D
485/. Epidemiological study for smoker said there is 10,000 person in the area , at start of the study
there is 2000 smoker, at the end of the study there is 1000 smoker, the incidence of this study is :
a) 10%
b) 12.5%
c) 20 %
d) 30%
Answer : ​B. New cases ÷ population at risk = 1000÷ 8000 * 100 = 12.5 %
486​​/ ​.In a cohort study on lubricant oil use and urinary bladder CA done over 20
years 10,000 exposed 10,000 non exposed 750 exposed got CA 150 non exposed
got CA Then they asked about the incidence in 1000 in one year?
A- 2.25
B- 45
C- .45
D- .225
Answer:​ A / In 20 y 20,000 p 900 Cases, so in one year 900/20=45 case for 20,000 population for 1000,
45/20=2.25
487/​​.Study about GDM association with malformation. 2 groups. 5000 control
group, 50 developed. 1000 diseased group 20 developed.(not sure about the
numbers) The best for risk factor association? (and calculate)
A- Relative risk =2
48

B- Odds ratio=2
C- RR=
D- OR =
Answer: ​B (By community specialist​)
488/​​Study we have 1000 DM 20 of them have congenital abnormal babies and
5000 control of non DM have 50 congenital abnormal babies... What determine
accuracy of the study?
A- OR 2.0
B- RR 2.0
C- OR 0.2
D- RR 0.2
Answer: ​A

489/What is the type of bias in meta-analysis?


A. Tendency to answer questions untruthfully or misleadingly on a survey
B. Recall retrospective studies.
C. Loss of follow up
D. Publication
Answer: ​D
490/​​Post test probability of a diagnostic test:
A. likelihood ratio
B. predictive value
Answer:​ A​
491/ ​A study aims at exploring the association of high fat intake and prostatic cancer., Group 1 has
prostatic cancer (1000) patients with 50 high fat intake, group 2 doesn't have cancer (1000) pt with 10
high fat intake
a) OR 0.52
b) OR 5.2
c) RR 0.52
d) RR 5.2
Answer: ​b​
492/​​ In a study they are selecting every 10th family in the city, what is the type of study?
A. Systematic random study
B. Stratified random study
C. Non randomized study
Answer: ​A
493/​​ Cohort study – 2 groups: one of them exposed to patient with positive MERSA and the other
group is not exposed to these patients. According to picture, what is the ratio of exposed to
unexposed? 
A. 66:1
B. 55:1
C. 44:1
Answer is : ​66:1 
**** there is two versions of this question in the collection, they are totally the same, but the number
aint so, watch out for them!! (the one with 600 total the correct answer is 66:1, the one with 500 the
correct answer is 55:1)
494/​​A cohort study of smoking and lung cancer was conducted in a small island population. There
were
49

a total of 1,000 people in the study, and the study was conducted over a ten year period. Four
hundred
were smokers and 600 were not. Of the smokers, fifty developed lung cancer. Of the non-smokers, 10
developed lung cancer. What is the Relative Risk? 
A. 7.5
B. 0.7
C. 8.5
Answer is: ​7.5
495/​​A study aims at exploring the association of cigarette smoking and the risk of IHD. Results were
as follows (Numbers are not exact, but are used for clarification purposes)
* Non smokers.OR:. 0.1
* Mild smokers..OR:. 1
* Heavy smokers.OR: 2
* Extensive heavy chain smokers OR:. 5
Which of these is true about this study findings? 
A. Dose-Dependent Relationship
B. Risk Association Relationship
Answer :​ A
** one new research question I couldn’t recall or even answer, they gave me different IC and RR(or OR
im not sure), in the choices and asked me, which study is the most precise or accurate or something like
that…
496/a young patient (paediatric age group), has a long history of PCP, aspergillosis and fungal
infection, what is the most likely diagnosis?
A. CD4 low count (or inactivity).
B. 22q mutation (digeorge syndrome)
Answer: ​B Digeorge syndrome characterizes with mentocrainial abnormality and hypoplasia of the
thymus, which makes the maturation and production of T cells is almost diminished​.
Should mention hypocalcemia.
497/​​ 8 years old child presented with painful limping, x-ray was given showing slipped capital femoral
epiphysis.
Answer: (​ I had two different questions showing the exact same finding on x-ray, easy to pick question,
and there is almost no need to know the scenario, just know the x-ray and the finding.) ​( Add the Ray )
498/​​Another case of limping child with history of limping but this time the femoral heads aren’t the
same size,
Answer a case of DDH​.
499/​​ Which of the following is synthesized in nucleolus? 
A - mRNA ..
B - rRNA ..
C - tRNA .. 
Answer is: ​B - rRNA ..
500/​​ Component of tetralogy of fallot:  
A. LV hypertrophy
B. Mitral regurgitation.
C. Septal Ventricular defect.  
  
Answer is: :​ C Septal Ventricular defect.
50

501/​​A 26 years old female presented with 34w gestation of age complaining of loss of her fetal
movement, she says that she noticed that since the morning but couldn’t come because she was in
work all day long, what is the next step?
A. -us
B. - non-stress test.
C. - vibration something.
D. -b-hcg
answer: ​B
502/ ​a 17 years old girl brought by her mother complaining of LL weakness and feeling of numbness
and tingling sensations, you examined the patient and found that she has no reflexes at all, and there
is marked reduced sensation, aside of that, the mother tells you that 2 weeks ago her daughter had
URTI, what's the dx?
answer: ​GBS​.
503/​​A very strange case, a paediatric age group boy was brought by his mother complaining that her
child is fatigued and irritable, he is also hyperactive at home, labs were given were completely normal
aside of microcytic hypochromic anemia (iron, TBIC, Ferritin all are within normal range). A peripheral
smear was obtained and BM biopsy was taken, showed something like hypermature or hypercellular I
( Bm biopsy shows erythroid hyperplasia and basophilic stipling ) what's the diagnosis?
A- I​DA
B- THALASEMIA.
C- SIDEROBLASTIC ANEMIA
D- SCA
Answer: ​ C

504/​​A young patient complains of blepharitis, she also has acne rosacea, what to give her?
a- Oral Doxycycline
b- Ceftriaxone
c- Topical doxy
d- Topical something.
Answer:​ A
505​​/​A mother complains of eye pain after injury of her daughters nail, with a picture showing cornea
with cells in the anterior chamber, what to give?
a- Topical steroid.
b- Antiviral.
c- Cover the eye (not literally was like this but I don’t remember the exact context)
Answer: ​C/ Topical abx, cycloplegic, cover, refere
506/ ​Male patient in his 40s, you did a colonoscopy and found 2 adenomas, when to scope him again?
a- 1 year
b- 3 years
c- 4 years
d- 5 years
answer: ​D
507/​​A case about a type of cancer, and you found keratinization in histology, what type of cancer is
this?
a- Adenocarcinoma
b- SCC
51

c- Melanoma
Answer: ​B
508/ ​Anti basement membrane dx?
Answer: GP syndrome
509/​​PSGN, what to give?
a. Diuretics for edema
b. Diuretics for HTN
answer: ​A
510/ ​MDD, lack of what?
Answer: ​serotonin
511/​​The most symptom of perforated Duodenal ulcer ?
Answer: ​Sudden epigastric pain
512/​​A young female presented with unilateral breast lump, she noticed it with the last two periods, it
hurt at periods times only, they gave the size too… … what to do?
( I don’t recall the answers tbh, but I do recall one) re-examine her after 2 months.
Answer: ​Observe - US imaging then FNA / Fibrocystic change Observe for 2-3 cycles Then FNA if persist
513/​​Young female patient, came with long history of alternating diarrhea and constipation, she has
been like this since 3 years but it got worse over the last week, you did a biopsy and found atrophic
villi, what to do?

A. steroids
B. immunosuppressants
C. gluten free diet
answer: ​c

514​​/​a kid, with ( I don’t recall the exact symptoms thou) labs showed hco3:40+
co2:20, na: low, k: 3.1, RBG: 26 what causes his symptoms?
A. -hydrogen malabsorption. (or absorption)
B. - sodium
C. - potassium.
Answer: ​sodium
:
515/​​An infant with facial purple lesion, what the organism?
A. -s.pneumonia
B. - Haemophilus influenzae
answer: ​Haemophilus influenzae
516/ ​a kid presented with knee pain for 2 days, it was swelled, red, and hot, aspiration of joint result
showed 75000 wbc, PMN: 85%, what to do next?
A. -oral antibiotic and discharge.
B. -pain medication.
C. -iv antibiotic as an outpatient for 14 days.
D. -admit, evacuate the joint and antibiotics.
Answer: ​D
517/​​Heavy alcoholic and heavy smoker, came with mid clavicular mass, what to do?
A. ⁃CT scan of the neck
B. ⁃ Excisional biopsy
C. ⁃ Needle biopsy
52

D. ⁃ Laparoscopy
Anwer​: ​A
518/​​which antipsychotic drug is related with metabolic syndrome?
A. - Olanzapine
B. - clozapine
Answer: ​A
519/ ​7 years old girl was brought by her mother, because she saw pubic hair, you examined the
patient and there's literally nothing (apart of fine hair in pubic area), what's the dx?
A. Normal
Answer: ​Premature adrenarche
520/ ​Antibody of cold agglutinin disease :
A. - IgE
B. - IgM
C. - IgG
D. - IgA
Answer: ​B
521/​​Pt with lung cancer with histopathology show: beast of polygonal cell and
granular eosinophils ... and somatostatin.
A. - squamous cell cancer
B. - Adenocarcinoma
C. - Hamartoma
D. - Carcinoid tumor
Answer: ​A
522/​​DM type 1 :
A. - CD4
B. - CD7
C. - CD10
D. - CD12
Answer : ​A
523/​​Pt with CHF and use digitalis , come with flashes and yellow vision, what the
cause?
- Digitalis toxicity
- Rental detachment
Answer : ​A
524/​​Child with jaw mass and lab show starry sky ?
A. - Hodgkin lymphoma
B. - Non-Hodgkin lymphoma
C. - Burkitt lymphoma
Answer : ​C
525​​/ ​Pt with stress incontinence and urethral detachment , what is the high
diagnostic test?
A. - Q-tip test
B. - Cystoscope
Answer:​ A
53

526/ ​Old Pt with rheumatoid arthritis and bilateral lower limb pigmentations ?
A. - pyoderma
B. - Stasis dermatitis
Answer :​B
527/ ​Infant with pic of rash in the neck fold , diagnosis?
Answer: ​erythrasma intertrigo
528/ ​Pt with white erythematous in plate, what’s treatment ?
A. - Tinidazole
B. - Miconazole
Answer : B
529/ ​Pt come from India with fever, splenomegaly and rash , diagnosis?
A. - leishmaniasis
B. - Tb
C. - Sarcoidosis
Answer : A
530/ ​What is pathological changing in Rapid Progressive GN ?
A. - crescent formation in bowman's capsule
B. - Dilated of proximal tubes
C. - IgA deposition in urine ..
Answer : A
531/ calculation of EDD
Answer : +1 year -3 month +7 days
532/ most common causes of colorectal cancer ?
Answer: Sporadic then Hereditary nonpolyposis ​colorectal cancer​​ (HNPCC)
Lynch syndrome — Lynch syndrome or HNPCC is an autosomal dominant syndrome
533-Increase insulin sensitivity ?
A. -lepton
B. Adiponectin
Answer:
534/ internal iliac ?
answer : Bladder
535-FHR drop to 80 ?
Answer: General anesthesia
536​​/ women on 40 wk on labor Since 3 hours the dilation inc.1 cm sane affecment ,
moderate contraction , pain 7/10 ?
A. -Oxytocin
B. -Rupture membrane
C. -epidural anesthesia continuous infusion
Answer: C
537/ SLE Pt have anemia and dec.platelet 90 , next ?
A. Platelet & RBC transfusions
B. AB
C. Steroid
D. Cyclosporin

Answer: C
54

538/ old age with back pain , fever , night sweating, acid fast bacilla + ve organism ?
Answer: Mycoplasma
539/RR
Answer: the ratio of the probability of an outcome in an exposed group to the probability of an
outcome in an unexposed group.
540/OD
Answer: statistic defined as the ratio of the odds of A in the presence of B and the odds of A
without the presence of B
541/incidence
measure of the probability of occurrence of a given medical condition in a population
within a specified period of time
542/ Old women 45 with heavy menstruation & every 2 week , low Hb , next ?
A. -endometrial biopsy
B. -blood transfusions
Answer: A/ vaginal US first
Note :
if medroxyprogesterone in choices its used to stop bleeding while planning for biopsy
If question included shocked then blood transfusion is given
If no progesterone in choices i would chose biopsy .
543/ most common cancer with PSOS?
Answer: Endometrial cancer
544/women with PSOS , she take progesterone for long time , no she stop progesterone
for ( weeks or months; not sure ) , she have risk of ?
A. -cervical dysplasia
B. -endometrial cancer
C. -osteoporosis
Answer: b
545-Thalassemia minor
A. increase A2 & F other difficult choices
Answer: Hg A 80-95% and normal Hb A2 3.5-7,5% and 1-5% hb F
546/​​ Hyperthyroidism case with signs and symptoms, what is the proper management of palpitation?
A. Propthy...
B. Propanol
Answer : B
​547/ ​Salmonella tx for ped
Answer: IM Ceftriaxone
548/ ​Acute pericarditis tx
Answer: NSAIDS /Aspirin

549/ ​Milestone of four color, dress by little help, Hope on one leg
Answer: 4 years
550/ ​Dm incidence new research
Answer :
551/ ​Crohn's location
Answer: Any part of GI but commonly ileocecal region, followed by the colon
552/ ​Height calculation of a boy
Answer: boy father + mother + 13 / 2
for girls father - 13 + mother / 2
55

553/ ​Acute appendicitis confirmatory modality


Answer: adult : CT, children and pregnant US

554/ ​Breast cancer with management and investigations


Answer:

555/ ​Ophtha pt with high blood pressure and on fundus exam showed exudates what will be the
needed diastolic
A. 110/100 in 12
B. 110/100 in 1-2 days
C. 90/80 in 12 h
D. 90/80 in 1-2 days
Answer: B

556​​/ ​ECG of WPW syndrome


Answer:

557/ ​Bee sting type of sensitivity


Answer: type 1

558/ ​Tamoxifen benefit on high risk pt


A. High
B. Moderate
C. Low
D. None
Answer: B ( moderate = Grade B, Grade A = High)

559/ ​Pt with meningitis symptoms neck stiffness, vomiting and “ vesicular rash” what is the type?
A. Aseptic
B. Pneumococcal
C. Tuberculous
D. Meningococcal
Answer: A

560/ ​Tx of pt with pharyngitis and is sensitive to amoxicillin what is the treatment?
A. Azytho
B. Ceftrixime
Answer: A ( If erythromycin its answer if not A )

561/ ​Pt came with PE what is the tx (duration was not mentioned)
A. IV heparin
B. IV TPA
C. IV streptokinase
D. Oral ...
Answer: A
Note : Stable .. A / Unstable .. C

562/ ​Pt has watery diarrhea after treating leg cellulitis? How would you treat this pt?
56

A. Metronidazole
B. Ceftriaxone
C. Ciprofloxacin
Answer: A

563/ ​Heroin withdrawal antidote


A. Naloxone
B. Methadone
Answer: B / Lofexidine

564/ ​Pt with symptoms of anemia and labs indicated anemia, what type of tx?
A. Oral ferrous sulphate
B. Iron .. cereals
Answer: A

565/ ​SCFE case


Answer:

566/ ​Case of peds with skin papules, culture indicated: gram negative diploma
What late complication pt will have?
A. Ataxia
B. Seizure
C. Hearing loss
D. Cranial nerve palsy
Answer: B
567/ ​CA 125 is marker for which type of ovarian cancer ?
A. ⁃ endothelium
B. ⁃ Germ cell
Answer : epithial
568/ ​ What is associated with increase risk of mortality in women (pregnant I think )
A. ⁃ phnochromcytoma
B. ⁃ Maternal age
Answer: A

569​​/​ Child with symptoms of URTI come to the ER due to dry cough increase at night preventing the
patient from sleep not relieved by elevation of the head in Examination there is fever

A. ⁃ croup
B. ⁃ epiglottis
C. ⁃ Bronchiolitis
D. ⁃ Asthma.
Answer: C

570/​​. ​ Child known case of Leukemia undergo bone marrow transplant his brother develop chicken
box what to give the child ?
A. ⁃ Varicella vaccines
B. ⁃ Varicella immunoglobulin
C. ⁃ Varicella vaccines & immunoglobulin
57

Answer: B

571/ ​ glipizide mechanism of action ?


Answer: Increase insulin secretion

572/​​ Asthmatic patient with migraine what to give as prophylactic

Answer: Amitriptyline

573/ ​. Elderly taken 75 mu of thyroxine for hypothyroidism for 6 months,he miss one dose .his lab
are normal what is the most appropriate dose by mcg
A. ⁃ 25
B. ⁃ 50
C. ⁃ 75
D. ⁃ 100
Answer: 75

574​​/ ​Man involved in RTA come t ER there was small wounds less than 2 cm X-ray show mid shaft
tibia fracture what to do next ?
A. ⁃ Reassurance & discharge
B. ⁃ Internal fixation with IM
Answer: Debridement irrigation EF

575/ ​ Child his X ray show greenstick fracture he is stable what to do ?


A. - discharges
B. ⁃ cast
Answer: B

576/​​Elderly with bone pain in examination there is bone tenderness especially the spine . labs
including high IgG & creatinine but low igA
A. multiple myeloma
B. Metastatic cancer
Answer: A

577/​​ Hemophilia father & hemophilia mother carriers what is the risk for their child to have the
disease ?
A. ⁃ 25
B. ⁃ 50
C. ⁃ 75
D. ⁃ 100
Answer: B

578/ ​ Patient in 10 week of pregnancy have vaginal bleeding she is known case of cervical inefficiency
with 3 previous miss carnage how to manage
A. ⁃ cervical cerclage
B. ⁃ Bed rest
Answer: B
58

​579/​​ Patient after dental extraction have numbness in his tongues which nerve affected
A. lingual
B. Chorda tympani
Answer: A / The ​lingual nerve​​ is a branch of the mandibular division of the trigeminal ​nerve

580​​/​ ​Young patient come with red eyes & tearing for 1 day no hx trauma , allergy no other associated
symptoms
A. ⁃ Reassurance
B. ⁃ Antibiotics
Answer: antihistamine

581/ ​ ​13 years old child his parents bring him to you concerning of his height & his sexual
development in examination height is normal his testes 4. Something cm no scrotal pigmentations &
fussy pubic hair

A. ⁃ Reassurance
B. ⁃ MRI
C. ⁃ Testosterone level
Answer:

582/​​. Hemophilia affected fathers & carrier mother what is the (repeated)
A. ⁃ 25
B. ⁃ 50
C. ⁃ 75
D. ⁃ 100
Answer: B

​583/ ​. Acute heart failure mechanism something about collagen change & BNP level
Answer: Bnp is very high in heart failure

584/ ​ Case alcoholic pancreatitis


Answer: GGT & AST + High MCV
585/ ​ Case about child abuse
Answer: Bruises of different/ different healing stages
586/ ​ Case about herpetic keratitis & corneal abrasion
Answer:
1- herpetic keratitis Topical therapy ganciclovir
2- corneal abrasion Topical Abx and cycloplegic and oral analesic
587/​​ Which anti dyslipidemia medication work by 3 hydroxy -3 methylglutaryl Co A reductase
inhibitors ( HMG-COA).
Answer: Statine
588/ ​Which antipsychotics less likely to cause TD ?
Answer: Clozapine
589/ ​Female with yellow vaginal discharge & multiple partner what the diagnosis?
Answer: Trichomoniasis
590/​​ Most common type of Vulvar cancer
A. ⁃ Adenocarcinoma
B. ⁃ Squamous cell carcinoma
59

C. ⁃ Adenosquamous carcinoma
Answer: B

591/ ​. Patient anemic receive hematic treatment come after 2 day complaining of dark stool
A. ⁃ iron dextran
B. ⁃ Ferrous sulfate
Answer:

​592/​​ Hemosiderin laden association with ?


A. ⁃ Rheumatic fever
B. ⁃ CMV
C. ⁃ Pneumocystis jiroveci
Answer:

593/​​. ​ Young female work as teacher have fear in the beginning of each class & she always think
some educational mistakes will occur soon or later
A. ⁃ specific phobia
B. ⁃ Mix phobia
C. ⁃ Social phobia
D. ⁃ Pain attack
Answer:

594/ ​. Young male afraid of Moro vehicle that increase Gradually he feel like he can not look at them
any mores and he have palpitations, & sweating
A. ⁃ PTSS
B. ⁃ Specific phobia
C. ⁃ Panic attack
Answer:

595/​​ ​Female have eczema because she can not stop washing her hands even if they are clean if she
did not wash them she will feel distress what is the treatment?
A. ⁃ Psycho- education
B. ⁃ Exposure & prevent response
C. ⁃ SSRI
Answer
596/​​. ​Child not paying attention during classes, not complete his home work or any task assigned to
him which defects he have ?
A. Attention
B. Hyperactive

Answer

597/ ​Pain on passive stretching of the foot, which muscle ?


Answer: Gastrocnemius

598/ ​Muscle that do dorsiflexion of the foot:


Answer: tibialis anterior
60

599/ ​Elderly lost his soupse after illness before 2 months, he has low mood, insomnia Dx?
A. Major depression
B. Minor depression
C. Cyclothymia
D. Bereavement
Answer:
600/ ​Parents brought their child with complain of eating clay and papers What to give
A. - Nothing
B. - Fluxiten
Answer:

601/ ​Patient with lung cancer, you want to see the lymph nodes involved or not, to decide the
treatment What is the best?
A. MRI
B. Perfusion
C. CXR
D. PET
Answer:
602/ ​Case of cholecystitis (with labs showing high amylase) Ttt?
A. - Conservative
B. - ERCP
C. - Lap chole
D. - GI Endoscopy
Answer:
603/ ​Pt received blood in 1975, he may get which type of hepatitis ?
A. Hep A
B. Hep C
C. Hep D
D. Hep E
Answer: ( No hep B in the option )

604/ ​Case of gastric mass


A. - US
B. - H. Pylori tests
Answer:
605/ ​Best to diagnose Meigs syndrome
A. - Tumor markers
B. - Histopath
C. - MRI
Answer:

606/ ​Boy had right eye trauma then the other eye also affected, after inoculation of the right eye
there is CD8 lymphocyte, what is the cause ? Or what’s happening
A. - Antigen release
B. - Lymphocyte ..
Answer:

607/ ​Innate immunity components?


61

A. Complement
B. Phagocytosis
C. inflammatory process
Answer:
608/ ​Elderly ​with​ acute urinary retention, best treatment is:
A. - Cystoscopy and TURP
B. - Semi-urgent prostatectomy
C. - Urethral cath and culture
Answer:

609/ A case about patient having symptoms and labs indicates gout …. Which of these
diuretics caused this problem -
A. thiazide
B. loop diuretics
C. spironolactone
610/ female complaining of. Infertility for 5 years and has a history of multiple Dilation
and curettage .... what's the diagnosis ?
Answer: Asherman syndrome

611/ 26 Y Female came to health clinic for URTI and she was diagnosed and given the
plan ,,, which of the screening test would you like to do right now : -
A. pap smear
B. uterine biopsy
C. mammogram

Answer:

612/ Ecg of atrial flutter …what the management maybe digoxin not sure .. please
check

613/ 80-100 more potent than morphine ... -fentanyl (correct answer) -tramadol

614/ Man is very sick, presents to ER with fever and tachypnea. After he is stabilized,
culture shows strep pneumonia. What is a possible complication?
A) Spesis
B) Meningitis
C) Pneumonia
D) Sinusitis

615/ What is correct regarding liquid method for pap smear?


A) specimen from transformation zone
B) specimen taken using a brush and spatula
(both correct)
62

616/ Healthy parents have a healthy daughter and a son with cystic fibrosis. They are
asking about the chance that their daughter is a carrier.
A) 1 out of 2
B) 2 out of 4
C) 1 out of 25
D) 1 out of 3
(exact answers given. A and B both 50% so idk...)

617/ Bacteroids fragilis after stab wound (no metronidazole in options):


A. Clindamycin
B. Ceftriaxone
C. Aztreonam

618/ Party, food poisoning 6 hours later. Vomiting, diarrhea, abdominal pain.
A) C. Perfrengins
B) S. Aureus

619/What causes production of exhaled nitric oxide in asthmatic?


A) Macrophages
B) Epithelium
C) smooth muscle cell
D) bronchi

620/Best research study for effectiveness of preventative measure for periodic health
check
A) RCT
B) Cohort
C) case control

621/ Pregnant woman 38 weeks gestation, healthy. Blood pressure 140/90 at antenatal
visit with no proteinuria. What is your management
A) diuretic
B) immediate culture and sensitivity
C) monitor BP regularly
D) give aspirin

622/ Woman presented to ER with 1 hour history of unilateral weakness and other stroke
symptoms. No headache. What is your immediate management
A) brain CT (should be the correct answer)
B) tPA
C) aspirin
63

623/ Patient presented to ER with apparent drug overdose. He was previously diagnosed
with depression and given citalopram. Anion gap 18 and metabolic acidosis. What did he
overdose on?
A) aspirin
B) citalopram
624/Which one has anti osteoclastic action ?
Asner: Bisphosphonate medications
625/How does the protein enter the Perexisome
A. Folded with N -terminal
B. Folded with C-terminal
C. Unfolded with C-terminal
D. Unfolded with N-terminal
626/Apoptosis triggered by
A. P35
B. BCL2

627/ Pregnant women had a C-section last pregnancy She's at the hospital , Now has twin
pregnancy 37 weeks of gestation ( one in breech position , the other in cephalic position)
. Had fluid discharge from vagina . Nitrazine paper showed blue green color , the doctor
decided to go c-section .Why c-section is indicated here?
A. Amniotic membrane rupture
B. Fetal position
C. Previous C-section

628/ Patient was playing barefoot , injured the sole with sharp glass , what’s the most
superficial structure injured in this patient ??
Answer: All were ligaments

629/ Child 3y/o with meningitis , what to give his older brother 5 y/o as a prophylaxis?
Answer: Rifampin

630/CTG showing late decelerations (decelerations after uterine contractions)


A. Umbilical cord compression
B. Uteroplacental insufficiency
C. Fetal head compression
64

631/ Measles case


Answer: Conjunctivitis, runny nose Classic rash and koplik buccal spots ( in pic )

632/ Middle age female with Oral and genital ulcers , cervical + inguinal lymphadenopathy
, vaginal discharge What's most likely diagnosis?
A. Herpes
B. Chancroid
C. Lymphedema venereum
D. Syphilis

633/ Bacterial vaginosis case asking about treatment:


Answer :metronidazole

634/ 15 y/o patient with pneumonia (x-ray picture shows lower lobe changes ) in right
lung What can you auscultate ?
A. Widespread Crackles in right hemithorax
B. Ronchi all over right hemithorax
C. Bronchial sound in the basal area in the right

635/ 55 y/o female , lase menses were 7 years ago . Shes having spotting bloody ,
specially after intercourse , whats most likely diagnosis?
A. Vaginal atrophy
B. Endometriosis
C. Endometrial cancer

636/ Patient woke up can't open his eyes , with purulent discharge from the eye and
crusts on the edges , what's the diagnosis?
A. Viral infection
B. Bacterial infection
C. Foreign body

637/ 50 y/o woman k/c of CKD with weakness in back and thigh muscles , her labs :
calcium decreased , ionized calcium decrease , elevated PTH and elevated Alkaline
phosphatase This woman at risk for developing which disease

A. Primary hyperparathyroidism
B. Osteoporosis
C. Osteopenia
D. Can’t remember last choice

638/ Patient had a nail injury in the cornea (with picture), what to do
A. Topical steroid ointment
B. Topical steroid drops
65

C. Topical antiviral drops

639/ Smoker with lung cancer , producing keratin ?


A. Squamous cell carcinoma
B. Adenocarcinoma
C. Other cancers in the options

640/ Skin lesion in genitals was biopsied and showed


Tree like growth with Keratosis and parakeratosis , how do you treat this conditions?
A. 5-flurouracil
B. Surgical resection
C. Can’t remember the rest

641/ Appendicitis case , appendix was removed in surgery , what can you see in biopsy ?
A. Neutrophils in muscularis layer
B. Giant multinucleated cells in muscularis layer

642/ Blepharitis and Rosacea , whats the treatment?


Answer :Many antibiotics were in the options including doxycycline

643/ Young female hx of mother died at young age , patient felt light headed when doing
exercises , holter ecg was done , no arrhythmia was noted average HR =65. Echo was
done showed septal thickening whats the best treatment for this patient?
A. Pacemaker
B. Septal myotomy

644/ Case of pericardial effusion on x-ray ( showing cardiomegaly)


645/ Atrial flutter ECG
646/ Many PVC in a symptomatic patient asking how to treat?
A. Propranolol
B. Verapamil
C. Other

647/ ECG with hyperkalemia

648/ Case with typical presentation of MI , ECG shows non ST elevated MI .

649/ Patient not on any medications with persistent hypertension 170/60 and
hypokalemia (potassium was 3 ) what's the diagnosis?
A. Essential hypertension
B. Primary Hyperaldosteronism
66

650/ Cystic fibrosis gene where is is located ( in which chromosome and which arm short
or long)
Answer: Long arm 7q31.2

651/ Neurofibromatosis type 1 ( in which chromosome and which arm located)


Answer:

652/ Patient has epistaxis when the heater at home is on , what to advice the patient?
A. Air humidifier
B. Ephedrine spray
C. Vitamin C

653/ Beri beri in young child asking what vitamin deficient ?


Answer:
654/ Type 2 hemorrhagic shock what to give?
A. Whole blood
B. Colloids
C. Crystalloids
D. RBCs and crystalloids

655/ Patient with bleeding he has decrease BP and increased HR , Which of the following
is decreased ?
A. Coronary blood flow
B. Venous capacity

656/ P value of 0.13 what does it mean ?

657/ 9 month old child what vaccines to give?

658/ HIV with kaposi sarcoma how to treat?


A. Chemo radiotherapy
B. Radiotherapy
C. Other options

659/ Patient with liver failure on central venous catheter had sepsis , culture showing
budding yeast ?Which antifungal to choose?
Answer: Caspofungin

660/ Most common injury in a newborn ?


A. Shoulder dislocation
B. Clavicular fracture
C. Hip dislocation
D. Don’t remember the last option
67

661/ Patient has mixed type hearing loss , her mother had the same , whats the
diagnosis?
A. Osteosclerosis
B. Acoustic neuroma

662/ Severe premenopausal syndrome how to treat ?


Answer: OCP

#########
663/ Thyroid carcinoma arise from parafollicular cells -medullary carcinoma
Answer : follicular carcinoma

664/ Non human milk fasting hours preoperative according to American society of
anaesthesia
Answer: - 2h>>> this for clear liquids -4h >> this for breast milk -6 h >> non human milk

665/ Pudendal nerve block … which of these area will be not affected
Answer: rectum

666/ What will prevent seizure in preeclampsia ....


-magnesium sulfate

667/ orbital pseudotumor...whats the management


A. -radiation
B. - surgical excision
C. -Irradiation
D. -systemic steroid
Answer: D

668/ Drop foot... which nerve injured


Answer: common peroneal nerve

669/ Sicklar with back pain… laps showed pancytopenia … which organism may cause
this condition ?
Answer: Parvovirus B19
670/ neurofibromatosis type 1 chromosome ?
Answer: 17q 11.2 long aram
671/ case about myasthenia gravis … what is the pathophysiology
Answer: ACH receptor antibody
68

672/ ethics Q… a doctor was invited to speak about investigation and management of
heart disease and the invitation from a 0/ Pap smear screening
681/Genital ulcers ( in q didn’t mentioned if it painless or painful!)
Pt primigravida everything normal , except fundal height higher then gestation age
Intro-ductal papilloma management
Patient on active labor fetal heart drop type of anesthesia : general
Common type of fibroid : intramural
Recurrent d&c : asherman
Recurrent abortion : anti phospholipids syndrome
Contraindication of instrumental delivery is : cephalopelvic disproportion
Ovarian mass and ascites : anterior dull lateral tympani
[8:09 PM, 11/12/2018] +966 59 933 2316: Ophthalmology nov 11
Chalazion pic common to be associated with : discharge , plephritis was not in the
options
Temporal arthritis : loss of vision management
Newborn past history of conjunctivitis doxycycline tt , now present with white patch on
the tongue : topical nystatin
Orbital Pseudo tumor management : steroid
Eye trauma : release of sequestrated antigen
Purulent eye discharge is : bacterial
69

* Penile ulcer + Lympadenopathy: Primary Syphilis (ttx: Penicillin G, diagnosed by DFM).


* Pregnant with hyperthyroidism: Propylthiouracil (anti-thyroid), or Propanolol if no
anti-thyriod(b-Blocker).
* Uncomplicated cystitis in a child: oral amoxicillin [Empirically]
* Thrichomonas Vaginalis ttx: Metronidazole.
* Asthma + BPH: Give a-Blockers (Tamsulosin, Terazosin or anything ending with -sin
-sin)
* Dementia treatment in general: Rivastigmine.
* End stage liver disease with fungal infection: Amphotericin (or Flumicatin if not in the
choices)
* Gonnohrea ttx: Ceftriaxone (espically if pregnant)
* Tinea pedis ttx: Topical antifungal.
* SLE complaining of erythema (mostly in face with burning sensation and
hyper-pigmentation) —> SLE Discoid.
* Patient complaining of recurrent red eye and tearing all her life: Conginital Glaucoma (If
not present choose congenital cataracts).
* Percussion to differentiate between ascites and ovarian cyst:
* Ovarian cyst percussion —> Dull in central, tympanic in peripheral.
* Pre-mature ovarian failure (i.e Pre-mature menopause): before 40 years.
* Prognosis of Schizophrenia: 50-60% bad prognosis (30% good prognosis).
* Alphafetoprotein screening in pregnant: 17 weeks.
* IUFGR/death suspected: Oligohydrominas.
* Patient came to ER overdosed (pinpoint pupil usually), improved with Naloxone: Opioid
toxicity (e.g morphine)
* Peptic ulcer disease: Epigastric pain (severe sudden in perforated duodenal ulcer)
* Lip lymph nodes:
* Middle lip —> Submental.
* Lateral lip —>Submandibular.
* Testes lymphatic drainage —> Para-aortic lymph nodes.
* Patient with inferior alveolar nerve injury, what is the symptom?:
Paresthesia/numbness/paralysis of lower lip and teeth.
* Loss of taste sensation in anterior tongue: Facial nerve (cordani)
* Patient can’t raise arm above shoulder (winging scapula): Long thoracic nerve injury.
[Root]
* PCOS increases risk of: Endometrial cancer.
* Bilateral breast cancer —> (Lobular)
* Mild-moderate BPH ttx: a-Blockers.
70

* Pregnant patient, on U/S snow-storm appearance: Complete mole or complete hydatid


multiform.
* Primary amenorrhea without uterus: Mullerian Dysgenesis.
* Patient with high Ca125 —> Ovarian cancer, especially epithelial type.
* Calculation of ovulation day: (Cycle days - 14)
* Liver zones affected by low blood supply: Central zone is the first to be affected.
Peripheral zone has highest blood supply.
* Female patient 40 years with breast lump. Mammogram and Ultrasound are normal:
Check after 2 cycles.
* Patient on Tamoxifen came with vaginal bleeding: Exclude endometrial cancer, because
Tamoxifen induces endometrial hyperplasia —> increase risk of malignancy.
* Young female with breast lump, what to do to confirm diagnosis: FNA/biopsy.
* Patient with 6 months dysmenorrhea with pain during intercourse and defecation, and
can’t conceive: Endometriosis.
* Hypoparathyroidism presentation: Decreased Ca —> Convulsions.
* Patient can’t extend hand after trauma: Radial nerve injury —> Spiral humerus.
* Theanar muscle atrophy: Median nerve injury.
* Winging scapula: Long thoracic nerve injury (from root).
* Appendix artery? —> Superior mesenteric artery —> Ilioceliac —> Posterior
appendicular artery.
* DeQuervain’s Tenosynovitis: Patient with wrist pain, positive Finkelstein’s sign: NSAIDs
if not effective —> Thumb and wrist spica/cast.
* Patient with eclampsia, given MgSO4 and her RR. What to give?:
* MgSO4 Toxicity signs: Absent/weak deep tendon reflexes, drop in RR and drop in HR.
* Patient after delivery, was given MgSO4, had bleeding. She had an increase in HR and
decrease in BP. What is the cause?: Hemorrhage (Not MgSO4).
* Patient with overdose in ER, comatose with dilated pupils: TCA overdose.
* NF mode of inheritance: Autosomal Dominant.
* NF1 gene: 17q11.2
* NF2 gene: 22.12.2
* Tall and thin patient, has laxity of joints. His mother has the same condition: Typical
Marfan Syndrome (Autosomal dominant).
* What is the cause of death in Marfan Syndrome?: Aortic rupture/aneurysm.
* Burkitt’s Lymphoma: C-MYC gene. Stary-sky appearance. From Ghana. Translumination
test: 8:14.
* Gene related to DM: HLA-DR4
* Gene related to RA: HLA-DR4
* Direct bilirubin (conjugated) increase: Post-hepatic —> stone/obstruction
* Indirect bilirubin (unconjugated) increase: Gilbert [Gene: UGT1A1](after stress like
infection or pregnancy, asymptomatic patient), hemolysis.
* Vanconi anemia: Autosomal recessive MOI.
* Hydrofetalis:
* Case of Gout, what is increased?: Uric acid/increase in purine in aspiration.
71

* Lactic acidosis in a young child: Pyruvate Carboxylase.


* Female with joint pain and swelling, on aspiration she had high WBCs without crystals:
Septic arthritis. Usually with high PMN, WBCs, fever.
* Systemic sclerosis [Dysphagia, stiff hands, Ryanoid’s phenomena.]: Anti-CL70.
* Most common cause of DKA in children —> Missed insulin dose.
* Most common cause of DKA in adults —> Infection.
* Milestone of driving a tricycle: 3 years.
* Milestone of drawing a straight line: 2 years.
* Best exercise for patient with coronary heart disease: Isotonic.
* Isotonic —> means cardio. Isometric —> means weight-lifting.
* Child with diarrhea and bad odor flatulance and abdominal distention: Gardiasis.
* Child has polydipsia and polyuria, and family has same problem: DM.
* Patient with 2 months history of throbbing headache when doing any work (playing
football, intercourse, etc): Excertional headache. (If with photophobia and others —>
Migraine)
* Old patient with headache relieved by paracetamol [Vitals: high BP] what is the cause?:
Essential HTN.
* Essential Tremor: Disappears at rest, patient not on drugs, when moving or reaching for
objects the tremor occurs.
* Patient with neck pain, more in the morning, X-ray was done to her and showed
osteophyte. Diagnosis: Ankylosing (cervical) Spondylosis. [Gene: HLA-B27]
* Patient with RA. Signs: morning stiffness, small joints involved except DIP, joint
deformity, bilateral nodules. Gene responsible: HLA-DR4.
* Polymalagia Rhuematica: Bilateral stiffness, pain and tenderness in shopulder.
[Differention from Polymyositis is + weakness]
* Polymyositis increases the risk of malignancy.
* Dematomyositis: “Rash” —> increases risk of malignancy.
* Screening for cervical cancer (Pap smear): Starts after 3 years of first intercourse/at age
21.
* Pap smear: 21-30 —> each 3 years.
* Pap smear: 30-65 —� �> each 5 years + HPV.
* Child with broken forearm picture with skin opening: Surgical debridement...
* Child came to ER with mass in inguinal area, severe pain and tenderness, mass
enlarging and negative cough impulse —> Strangulated hernia.
* HIV pregnant decreased from 400 to 200 —> SVG.
* Vaccine that is not given if patient or any contact is immunocompromised: Oral Polio
Vaccine (OPV).
* Patient came to ER with vertigo, agitation, nausea and vomiting: Check alcohol level
—> Blowing test.
* Patient feels ill, was diagnosed with choriocarcinoma. What is resposible for these
symptoms?: bHCG.
* Patient had injurt to lateral side of knee, had pain and swelling in medial side: Valgus
test positive —> Medial collateral ligament injury.
72

* Varus test is positive —> Lateral collateral ligament injury.


* HIV patient refuses to tell his wife: Tell authorities.
* Treatment of urge incontinence —� �> Oxubutrin (MOA: Anti-cholinergic OR Inhibit
muscarinic receptors).
* Patient treated for meningitis then came with low platelet, low Hb, low WBC
(pancytopenia). Which of the following is responsible for this presentation:
Cloramphenicol.
* Sulfa drug reaction/allergy —> Blisters.
* OCP that cause increase in K —> Drospirenone.
* Drug that can cause acne —> “Lefornoldesterol”??
* Patient with bloody diarrhea, hematuria, petechial rash (with change in consciousness
or CNS involvement), low platelet and high creatinine: TTP.
* TTP treatment —> Plasma echange.
* TTP gene —> ADAMT13.
* DM patient lower limb changes: pallor, weak pulse —> Arterial insufficiency.
* Study on the effect of drug A and B blah blah the disease rate went down etc —>
Effectiveness.
* Interferon Gamma —> Helps increase phagocytosis killing.
* Patient came to ER, can’t feel the apex, diminished/muffled heart sound, distended neck
veins and low blood pressure. —> Cardiac tamponade.
* Cardiac tamponade treatment —> Pericardiocynthesis (immediately).
* Pregnant in delivery, can feel the nose, chin and orbital ridge of baby. —> Face
presentation. (Induction with Oxytocin and SPV).
* Brow presentation —> can feel nose, eye and forehead.
* Abnormal pap smear, unable to take biopsy by colposcopy. —> Cone biopsy.
* Milestone of social smile: 2 months.
* rRNA —> Made in nucleolus.
* mRNA —> Made in nucleus.
* Generation of energy in and out of the cell —> GTP Cytoplasm
* AML treatment:
* Gastric ulcers —> ABO: A
* Duodenal ulcers —> ABO: O
* Treatment of PCOS —> Weight loss, OCPs, Spironolactone, Clomiphene citrate,
Letrozole.
* Uterine fibroids risk factors —> 1. Age, 2. African race
* Anti-acid that cause constipation —> Aluminum.
* DIP nodule —> Heberden's nodes.
* PIP nodule —> Bouchard's nodes.
* Osteoporosis Rx: ATP-based Biphosphonates —> Etidronate, Tildronate, Clodironate
(ending in -onate -onate).
* Aspirin rate of protection from stroke to risk of developing ulcer —> 12%
* Gilbert's —> UGT1A1 gene
* Ankylosing Spondylitis —> HLA-B27
73

* Risk of having another baby with cleft palate —> 4%


* Prophylaxis of cluster headache —> Verapamil
* Cluster headache —> 100% O2 mask.
* “Transmural inflammation” —> Crohn's Disease.
* Hypothyriodism/DM —> Adhesive Capsulitis: Shoulder pain, increased at night,
decreased ROM.
* Graph used to describe pretest post-test probability —> Nomogram.
* Hepatospleenomegaly + Coombs Test (+) —> CLL.
* Urate stones prophylaxis —> Citrate Na.
* Metoclopramide toxicity antidote —> Benzotropine / Diphenhydramine.
* Bachet's Disease Mx —> Steriods —> Colchicine (if steroid-resistant).
* Colle's fracture —> Child fell on outstretched hand.
* “Popcorn Calcifications” on CXR —> Hamartoma.
* RSV lung infection Rx —> Ribavarin.
* CTG Conditions:
* Early decelerations —> Head compression.
* Variable decelerations —> Cord prolapse.
* Late decelerations —> Placental insufficiency.
* E. Histolytica —> “Flask-shaped ulcers” in colon.
* SLE Dx —> (+) ANA, (+) dsDNA.
* “Roth Spot” —> Supports Infective Endocarditis diagnosis.
* OSA —> Indication for Tonsillectomy.
* Blood film shows “Inclusion bodies” or “Pencil Cells” or “Holly Jolly Bodies” —> SCA.
* Hypothyroidism patient wants to get pregnant —> Decrease dose by 30%.
* Risk of Schizophrenia if father has it —> 10%.
* SSRI that usually causes overdose —> Citalopram.
* SSRI that usually causes weight gain and ED —> Sertraline (Line= Erection)
* Asthmatic with itching of the eye —> Vernal conjunctivitis.
* Vessels swollen in hemorrhoids —> Veins.
* Active form of Thyroid hormone —> T3.
* Status Epilepticus —� �> Benzodizepam —� �> IV Phenytoin.
* Bicurnuate Uterus —> Breech presentation.
* Bilateral cellulitis —> Venous stasis cellulitis.
* Giving Rogham in Rh- mother —> 24 weeks - 72 hours after delivery.
* Raynaud’s phenomena: Discoloration on hand fingers —> white, blue then red.
* New recommendation for Infantile Sudden Death Syndrome: Pacifier.
* Cherry red spots in Retina —> Retinal artery occlusion.
* Wrenkle’s Area damage leads to —> Fluent Aphasia (Receptive)
* Baby born with Apgar score 3 —> Warming and drying.
* Sideroplastic anemia: Microcytic hypochromic, with “ “ on blood film. [Child eating toys
and wall paint]
* Legg-Calvé-Perthes Disease: Hip pain, limping, fixed flexion and internal rotation.
* Vitamin B12 deficiency —> Can present as clinical dementia.
74

* CXR: “Coin lesion surrounded by crescent shape” —> Aspergillosis.


* Mumps complication —> Meningitis.
* Otitis Media —> Most common cause for brain abscess.
* Most common brain tumor in adults: Astrocytoma.
* a1-Agonist (Epinephrin) drops —> Contraindicated in Glaucoma.
* Face presentation that can be delivered vaginally —> Mento-anterior.
* Intradermal combined contraceptive —> Increases risk of DVT.
* “Holly Jolly bodies” —> Blood smear of SCD.
* Picture of spoon-shaped nail —> IDA.
* Prostatic cancer marker —> PSA (OR reverse it: Acid Phosphatase)
* Renal transplant rejection. Cell responsibe? —> CD8.
* Heroin withdrawal symptoms —> Give Methadone.
* Chromosome in DM2 —> Chromosome 20. [And HLA-DR4]
* Kleinfilter Syndrome: XXY, enlarged breasts male and no pubic hair.
* Child with leg pain during night only —> Normal growth.
* Patient with mania, agitated and you want to calm him down —> Haloperidol. For very
dangerous patients —> Restraints.
* Strongest risk factor for placental abruption —> Previous abruption.
* Lichen Planus commonest site —> Mouth (and mucus membranes). Then Wrists,
ankles, nails and scalp.
* Antibiotic that causes rash —> Vancomycin.
* Splenic artery ligation will affect which part of stomach? —> Fundus.
* External laryngeal nerve Injury will affect —> Tension of vocal cord.
* Recurrent laryngeal nerve Injury will affect —> Esophagus.
* Anti-psychotic that causes brown discoloration of the eye —> Thioridazine (close to
“Thiazide”)
* Hyperthyroidism S/S, on FNA there was “Hurthel Cells” what’s the Dx —> Follicular
Thyroid Cancer.
* What about Papillary? —> [ ]
* Colorectal cancer screening recommendation —> 50-75 Years.
* Antipsychotic with extremely low risk of Tardive Dyskinesia —> Clozapine.
* PTDS best short-term management —> Lorazepam.
* Best treatment for Anxiety —> Ecitalopram.
Infective endocarditis treatment?

1-multiple antibiotics for short time


2-multiple prolonged bacteriostatic agents
3-multiple extensive bactericidal agents for long time 🤙🏼
4-surgical interventions for all patients

Ringworm stain:
India ink
Ziehl–Neelsen
75

Gram stain
KOH

Bahcet’s disease oral ulcer tried corticosteroid mouth wash and corticosteroid paste:
interferon alpha
‫وﻣﺮه ﺣﺼﻠﺘﻬﻢ ﻣﺠﺎوﺑﯿﻨﻪ اﺳﻢ ﻛﺬا‬
Azin....

Questions that need to be confirmed

N Number of Q Page Major

1 38 3 infectious

2 58 4 hematology

3 78 6 hematology

4 83 7 OB

5 97 9 GS

6 99 9 Medicine /GI

7 136 13 Medicine

8 147 14 pedia/neonate

9 155 15 pedia/Gentic

10 181 19 Basic /pharma

11 188 20 ortho

12 192 20 ortho

13 209 22 psychrity

14 217 23 medicine/Rheumatology

15 221 23 infectious

16 225 24 Pediatirc
76

17 251 27 medicine/rehabilitation

18 286 30 OB

19 288 31 ENT

20 295 32 OB

21

22

23

24

25

Note: pictures left + 3 files in Email compare between each one

Das könnte Ihnen auch gefallen